diff --git "a/math_hard_ru - Лист1.csv" "b/math_hard_ru - Лист1.csv" new file mode 100644--- /dev/null +++ "b/math_hard_ru - Лист1.csv" @@ -0,0 +1,240 @@ +task,solution,short answer,class,grade +"Девять действительных a1, a2, ..., a9 образуют арифметическую прогрессию. Известно, что a9 в 3 раза больше среднего арифметического этих девяти чисел. Найдите a1, если известно, что a4 = 6.","Пусть 𝑎 — первый член прогрессии, а 𝑑 — её разность, тогда девять членов прогрессии равны 𝑎, 𝑎 + 𝑑, 𝑎 + 2𝑑, …, 𝑎 + 8𝑑. +Среднее арифметическое чисел в арифметической прогрессии, состоящей из нечётного числа членов, равно среднему из этих чисел, т. е. в данном случае 𝑎 + 4𝑑. +Получаем уравнение 𝑎 + 8𝑑 = 3(𝑎 + 4𝑑), откуда следует 𝑎 + 8𝑑 = 3𝑎 + 12𝑑 и 𝑎 = −2𝑑. Тогда 6 = 𝑎4 = 𝑎 + 3𝑑 = 𝑑. Значит, 𝑎1 = −2𝑑 = −12.",-12,school,11 +"В понедельник у Семёна был день рождения, ему подарили некоторое количество рублей. Он решил не тратить все деньги сразу. Со вторника по субботу он тратил каждый день по 20 % от текущей суммы. Сколько рублей он потратил в четверг, если в пятницу его траты составили 384 рубля?","Обозначим через x количество рублей, которое потратил Семён в четверг. От имеющейся суммы он тратил 20% и оставлял на следующий день 80%, поэтому на пятницу у него осталось 4x рублей. В пятницу он рублей потратил 20% от 4x, то есть 4/5x , что составляет 384 рублей. Тем самым получаем, что в четверг Семён потратил x = 5/4 * 384 = 480 рублей.",480,school,8 +"Учитель написал на доске четыре различных целых числа. Отличник Паша перемножил какие-то три из них и получил 37, а отличник Ваня перемножил какието три из них и получил 74. Какое наименьшее значение может принимать сумма четырёх чисел на доске?","Заметим, что число 37 является простым, и получить его произведением трёх целых чисел можно лишь, перемножая числа 1, −1 и −37. Чтобы получить произведение 74, надо выбрать два множителя из набора {1, −1, −37} и один новый множитель. Тогда получается, что есть 3 варианта для последнего множителя: 1) 1 * (−1) * (−74) = 74, +2) 1 * (−37) * (−2) = 74, +3) (−1) * (−37) * 2 = 74. Очевидно, что наименьшая сумма всех четырёх чисел будет в случае, когда четвёртое число на доске равно −74, и она равна 1 + (−1) + (−37) + (−74) = −111.",–111,school,8 +"За год каждый из восьмиклассников гимназии № 1 получил по алгебре либо 8, либо 10 оценок (все оценки — от 2 до 5). Известно, что у любых двух восьмиклассников средние баллы по алгебре за год различны. Какое наибольшее количество восьмиклассников может быть в этой гимназии? Средний балл — это сумма всех оценок ученика, делённая на их количество","Для начала поймём, какие числа могут быть средним арифметическим как 8, так и 10 натуральных чисел. Обозначив соответственно сумму 8 чисел за 𝑛, а сумму 10 чисел за 𝑚, получаем равенство 𝑛/8 = 𝑚/10 . Домножив на 40, получаем 5𝑛 = 4𝑚. В частности, 𝑛 должно делиться на 4, а значит, величина 𝑛/8 — целая или полуцелая. Сумма 8 оценок может равняться любому натуральному числу от 16 до 40, т. е. при 8 оценках возможен максимум 25 различных средних баллов. Также сумма 10 оценок может равняться любому натуральному чис��у от 20 до 50, т. е. при 10 оценках возможен максимум 31 различный средний балл. Всего получаем 25 + 31 = 56 значений, но из этого количества нужно вычесть количество средних баллов, +посчитанные дважды. Как говорилось ранее, это целые или полуцелые числа: 2; 2,5; 3; 3,5; 4; 4,5; 5 (ясно, что каждое из этих чисел может быть средним баллом и при 8 оценках, и при 10; также средний балл всегда не меньше 2 и не больше 5). Получаем, что наибольшее количество различных средних баллов (оно же наибольшее количество восьмиклассников в этой гимназии) равно 56 − 7 = 49.",49,school,8 +"По кругу стоят 36 детей, каждый из них одет в красную или синюю кофту. Известно, что рядом с каждым мальчиком стоит девочка, а рядом с каждой девочкой стоит человек в синей кофте. Найдите наибольшее возможное количество девочек в красных кофтах.","Заметим. что не найдётся 3 стоящих подряд девочек в красных кофтах (иначе для средней из них не выполняется условие). Разбив 36 детей на 12 троек, получаем, что в каждой из них не более 2 девочек в красных кофтах, а всего девочек в красных кофтах не больше 2 * 12 = 24. Пример построить легко: в каждой из 12 троек по часовой стрелке располагаются две девочки в красных кофтах, а следом за ними мальчик в синей кофте. Ясно, что все условия задачи выполняются. +",24,school,8 +"Из города в деревню выехал автомобиль, одновременно с ним из деревни в город выехал велосипедист. Когда автомобиль и велосипедист встретились, автомобиль сразу же развернулся и поехал обратно в город. В итоге велосипедист приехал в город на 35 минут позже автомобиля. Сколько минут затратил велосипедист на весь путь, если известно, что его скорость в 4,5 раза меньше скорости автомобиля?","Отметим деревню 𝐴, город 𝐵, точку 𝑃 встречи автомобиля и велосипедиста, а также точку 𝑄, где оказался велосипедист в момент возвращения автомобиля в город. Поскольку скорости автомобиля и велосипедиста различаются в 4,5 раза, то 𝐴𝑃 / 𝑃𝐵 = 1 / 4,5 = 2 / 9. Поскольку автомобиль потратил на перемещения 𝐵 в 𝑃 и 𝑃 в 𝐵 одинаковое время, то и велосипедист потратил на соответствующие перемещения 𝐴 в 𝑃 и 𝑃 в 𝑄 одинаковое время. Следовательно, 𝐴𝑃 / 𝑃𝑄 / 𝑄𝐵 = 2 / 2 / 7. Поскольку велосипедист потратил на перемещение 𝑄 в 𝐵 ровно 35 минут, то на всё перемещение 𝐴 в 𝐵 он потратил пропорциональное время: 35 * 11 / 7 = 55 минут.",55,school,8 +"Паша выписал в порядке возрастания все натуральные делители натурального числа 𝑘 и их пронумеровал: первый, второй, …. Паша заметил, что если шестой делитель умножить на тринадцатый делитель, то получится исходное число 𝑘. Сколько натуральных делителей имеет число 𝑘?","Пусть натуральные делители числа 𝑘 упорядочены так: 1 = 𝑑1 < 𝑑2 < … < 𝑑6 < … < 𝑑13 < … < 𝑑𝑚−1 < 𝑑𝑚 = 𝑘. Заметим, что числа 𝑘 = 𝑘 𝑑1 > 𝑘 𝑑2 > … > 𝑘 𝑑6 > … > 𝑘 𝑑13 > … > 𝑘 𝑑𝑚−1 > 𝑘 𝑑𝑚 = 1 также являются делителями числа 𝑘, они различны, и их столько же. Значит, это те же самые числа, только в обратном порядке. Получаем, что 𝑑1 = 𝑘 𝑑𝑚 , 𝑑2 = 𝑘 𝑑𝑚−1 , … , 𝑑𝑚 = 𝑘 𝑑1 . Таким образом, делители разбиваются на пары «противоположных», дающих в произведении исходное число 𝑘: 𝑘 = 𝑑1 * 𝑑𝑚 = 𝑑2 * 𝑑𝑚−1 = … В каждой такой паре сумма индексов делителей равна 𝑚 + 1. Поскольку по условию 𝑑6 * 𝑑13 = 𝑘, получаем, что 𝑚 = 6 + 13 − 1 = 18.",18,school,8 +"На острове живут рыцари, которые всегда говорят правду, и лжецы, которые всегда лгут. Однажды собрались 10 жителей острова, все они надели на себя футболки с номерами от 1 до 10 (у разных жителей разные номера). Каждый из них сказал одну из фраз: «Среди собравшихся нет рыцаря, номер футболки которого больше моего», «Среди собравшихся нет лжеца, номер футболки которого меньше моего». Известно, что каждая из этих фраз прозвучала ровно 5 раз. Сколько рыцарей могло быть среди этих 10 жителей? Укажите все возможные варианты.","Рассмотрим людей, сказавших первую фразу. Среди них не более одного рыцаря (в ином случае рыцарь с наименьшим номером среди них соврал бы). Таким образом, всего рыцарей не больше 6. Также среди всех присутствующих есть хотя бы 1 рыцарь (в ином случая все лжецы, говорившие первую фразу, говорили бы правду). Для каждого количества рыцарей от 1 до 6 существует пример. Пусть люди говорят фразы в порядке их номеров футболок. Запишем в ряд номера произнесенных ими фраз: +• 2, 2, 2, 2, 2, 1, 1, 1, 1, 1: всего 6 рыцарей с номерами 1—5 и 10. +• 2, 2, 2, 2, 1, 2, 1, 1, 1, 1: всего 5 рыцарей с номерами 1—4 и 10. +• 2, 2, 2, 1, 2, 2, 1, 1, 1, 1: всего 4 рыцаря с номерами 1—3 и 10. +• 2, 2, 1, 2, 2, 2, 1, 1, 1, 1: всего 3 рыцаря с номерами 1—2 и 10. +• 2, 1, 2, 2, 2, 2, 1, 1, 1, 1: всего 2 рыцаря с номерами 1 и 10. +• 1, 2, 2, 2, 2, 2, 1, 1, 1, 1: всего 1 рыцарь с номером 10. +",123456,school,8 +"На клавиатуре компьютера Пети неисправна одна клавиша с некоторой цифрой (все остальные клавиши работают хорошо). Неисправная клавиша срабатывает только на каждое второе нажатие. Например, в случае неисправной клавиши «2» при вводе числа 12125252 получится число 112552. Петя попробовал ввести 10-значное число, но на экране появилось 7-значное число 7479189. Клавиша с какой цифрой могла быть неисправна? Укажите все возможные варианты.","Не напечатались 10 − 7 = 3 цифры. Это означает, что на неисправную клавишу нажимали хотя бы 5 раз, при этом не сработали первое, третье и пятое нажатие, но точно сработали второе и четвёртое. Цифр, встречающихся хотя бы дважды, на экране ровно две: 7 и 9. Ясно, что клавиша с 7 могла быть неисправной, например, в случае ввода числа 7774779189, а клавиша с 9 могла быть неисправной, например, в случае ввода числа 7479991899",79,school,8 +"В классе учатся 29 школьников: несколько отличников и несколько хулиганов. Отличники всегда говорят правду, а хулиганы всегда врут. Все ученики этого класса сели за круглый стол. Несколько учеников сказали: «Рядом со мной ровно один хулиган». Все остальные ученики сказали: «Рядом со мной ровно два хулигана». Какое наименьшее количество хулиганов может быть в классе?","Если бы по кругу нашлись три отличника подряд, средний из них точно сказал бы неправду. Следовательно, среди любых трёх подряд идущих человек должен быть хотя бы один хулиган. Выберем произвольного хулигана. Дадим ему номер 29, а всех следующих за ним по часовой стрелке людей пронумеруем числами от 1 до 28. Поскольку в каждой из непересекающихся групп (1, 2, 3), (4, 5, 6), …, (25, 26, 27), (29) есть хотя бы один хулиган, то всего хулиганов хотя бы 27/3 + 1 = 10. Заметим также, что хулиганов могло быть ровно 10. Опять же, пронумеровав людей по часовой стрелке числами от 1 до 29, пусть ученики с номерами 3, 6, 9, 12, 15, 18, 21, 24, 27, 29 — это хулиганы, а ученики со всеми остальными номерами — отличники. При этом все отличники, кроме отличника с номером 28, сказали первую фразу, а отличник с номером 28 и все хулиганы сказали вторую фразу. Несложно видеть, что все условия задачи выполняются.",10,school,8 +"Точки 𝐷 и 𝐸 отмечены соответственно на сторонах 𝐴𝐶 и 𝐵𝐶 треугольника 𝐴𝐵𝐶 так, что 𝐴𝐷 = 𝐸𝐶. Оказалось, что 𝐵𝐷 = 𝐸𝐷, ∠𝐵𝐷𝐶 = ∠𝐷𝐸𝐵. Найдите длину отрезка 𝐴𝐶, если известно, что 𝐴𝐵 = 7 и 𝐵𝐸 = 2.","Заметим, что треугольники 𝐷𝐸𝐶 и 𝐵𝐷𝐴 равны. Действительно, 𝐷𝐸 = 𝐵𝐷, 𝐸𝐶 = 𝐷𝐴 и ∠𝐷𝐸𝐶 = 180∘ − ∠𝐵𝐸𝐷 = 180∘ − ∠𝐵𝐷𝐶 = ∠𝐵𝐷𝐴. Отсюда следует, что 𝐷𝐶 = 𝐴𝐵 = 7 и ∠𝐷𝐶𝐸 = ∠𝐵𝐴𝐷 (рис. 3). Из последнего равенства углов следует, что треугольник 𝐴𝐵𝐶 является равнобедренным, 7 = 𝐴𝐵 = 𝐵𝐶 = 2 + 𝐸𝐶, откуда получаем 5 = 𝐸𝐶 = 𝐴𝐷 и 𝐴𝐶 = 𝐴𝐷 + 𝐷𝐶 = 5 + 7 = 12.",12,school,8 +"На доске были написаны числа 1, 2, 3, … , 235. Петя стёр несколько из них. Оказалось, что среди оставшихся чисел никакое не делится на разность никаких двух других. Какое наибольшее количество чисел могло остаться на доске?","На доске могло остаться 118 нечётных чисел: любое из них не делится на разность никаких двух других, потому что эта разность чётна. Предположим, могло остаться хотя бы 119 чисел. Рассмотрим 118 множеств: 117 пар (1, 2), (3, 4), (5, 6), …, (233, 234) и одно число 235. По принципу Дирихле в одном из множеств осталось хотя бы два числа. Это означает, что среди оставшихся чисел найдутся два последовательных, но тогда их разность, равная 1, является делителем любого другого оставшегося числа. Противоречие.",118,school,8 +"В таблице 3 * 3 расставлены действительные числа. Оказалось, что произведение чисел в любой строке и любом столбце равно 10, а произведение чисел в любом квадрате 2 * 2 равно 3. Найдите число, стоящее в центральной клетке.","Обозначим числа в квадрате слева направо: пусть 𝑎, 𝑏, 𝑐 — числа в первой строчке, 𝑑, 𝑒, 𝑓 — во второй строчке, 𝑔, ℎ, 𝑖 — в третьей строчке. Заметим, что 𝑒 = (𝑎𝑏𝑑𝑒 * 𝑏𝑐𝑒𝑓 * 𝑑𝑒𝑔ℎ * 𝑒𝑓ℎ𝑖) / (𝑎𝑏𝑐 * 𝑑𝑒𝑓 * 𝑔ℎ𝑖 * 𝑏𝑒ℎ * 𝑑𝑒𝑓) = 3**4 / 10**5 = 0,00081.",0.00081,school,8 +"Вася загадал двузначное число, а затем приписал к нему слева цифру 1, а справа — цифру 8, отчего число увеличилось в 28 раз. Какое число мог загадать Вася? (Найдите все варианты и докажите, что других нет.)","Пусть Вася загадал число n. При приписывании справа цифры 8 оно превращается в число 10n+ 8, а при приписывании слева цифры 1 оно увеличивается ещё на 1000, поэтому 10n + 1008 = 28n; 18n = 1008; n = 56.",56,school,8 +"В классе за каждой партой сидят двое учеников. Парт, за которыми сидят двое мальчиков, вдвое больше, чем парт, за которыми сидят две девочки. А парт, за которыми сидят две девочки, вдвое больше, чем парт, за которыми сидят мальчик с девочкой. Сколько в классе мальчиков, если известно, что там 10 девочек?","Пусть парт, за которыми сидят мальчик с девочкой, x. Тогда парт с двумя девочками 2x. Тогда всего девочек 2 * 2x + x = 5x = 10, откуда x = 2. Тогда парт с двумя мальчиками 4x = 8. Значит, всего мальчиков 2 * 8 + 2 = 18.",18,school,8 +"На доске записано натуральное число. Николай заметил, что может двумя способами приписать к нему цифру справа так, чтобы полученное число делилось на 9. Сколькими способами он может приписать к данному числу цифру справа так, чтобы полученное число делилось на 3?","Заметим, что разность между двумя числами, «замеченными» Николаем, меньше 10, но при этом делится на 9. Значит, эта разность равна 9. Это возможно только если приписанные цифры есть 0 и 9. Тогда легко видеть, что для делимости на 3 помимо этих двух цифр можно приписать также цифры 3 и 6. Итого 4 способа",4,school,8 +"Архипелаг состоит из нескольких малых островов и одного большого. Было решено построить мосты между островами так, чтобы большой остров соединялся с каждым малым островом двумя мостами, а любые два малых острова были соединены одним мостом. К 1 ноября были построены все мосты между малыми островами и несколько (не менее одного) мостов, ведущих на большой остров, — всего 28 мостов. Сколько всего островов в архипелаге?","Занумеруем малые острова архипелага. Если мост соединяет острова с номерами a и b, запишем на этом мосту меньшее из этих двух чисел. Предположим, что число малых островов в архипелаге не более шести. Тогда мостов с номером 1 не более 5, мостов с номером 2 не более 4 и т. д. На большой остров ведет не более 12 мостов. Тогда всего решено построить не больше, чем 5 + 4 + 3 + 2 + 1 + 12 = 27 < 28 мостов, что противоречит условию. Предположим, что число малых островов в архипелаге не меньше 8. Тогда мостов с номером 1 не менее 7, мостов с номером 2 не менее 6 и т. д. К большому острову построено не менее двух мостов, поэтому общее число построенных мостов не менее 7 + 6 + 5 + 4 + 3 + 2 + 1 + 2 = 30 > 28, что также противоречит условию. Значит, малых островов в архипелаге 7, а всего островов — 8",8,school,8 +"В мешке у Деда Мороза находятся меньше ста подарков для Пети, Вася, Бори и Лёши. Дед Мороз отдал половину подарков Пете, пятую часть — Васе, седьмую часть — Боре. Сколько подарков досталось Лёше?","Чтобы Дед Мороз мог отдать половину подарков Пете, общее количество подарков в его мешке должн�� делиться на 2. Также, поскольку он отдал пятую часть Васе, а седьмую часть Боре, общее количество подарков должно делиться на 5 и на 7. Таким образом, количество подарков должно делиться на НОК(2, 5, 7) = 2 * 5 * 7 = 70. По условию задачи количество подарков меньше ста, поэтому их может быть только 70. Тогда Пете он отдал 70 / 2 = 35 подарков, Васе — 70 / 5 = 14 подарков, а Боре — 70 / 7 = 10 подарков. Таким образом, Лёше он отдал 70 − 35 − 14 − 10 = 11 подарков.",11,school,8 +"Рыцарский турнир длится ровно 7 дней. К концу четвертого дня сэр Ланселот не успел сразиться лишь с одной четвертью от общего числа участников турнира. А сэр Тристан к этому времени сразился ровно с одной седьмой из тех рыцарей, с кем успел сразиться сэр Ланселот. Какое минимальное количество рыцарей могло участвовать в турнире?","Пусть Ланселот не сразился с x рыцарями. Тогда общее число рыцарей равно 4x, а сразился Ланселот с 3x − 1 рыцарем (общее количество за вычетом x и самого Ланселота). Тогда Тристан сразился с 3x−1 7 рыцарей. Чтобы найти наименьшее возможное количество рыцарей, необходимо подобрать минимальное x такое, что 3x−1 делится на 7. Значения x = 1, 2, 3, 4 не подходят, а x = 5 подходит. Таким образом, наименьшее возможное число рыцарей равно 20",20,school,8 +"Володя расставил несколько (возможно 0) шахматных фигур на доску 8 × 8. Лёня заметил, что в каждом квадрате 2 × 2 стоит одинаковое количество фигур. А Влад заметил, что в каждом прямоугольнике 3 × 1 (или 1×3) стоит одинаковое количество фигур. Сколько фигур было выставлено на доску? (Укажите все варианты и докажите, что других нет)","Предположим, что в каждом квадрате 2*2 стоит m фигур, а в каждом прямоугольнике 1*3 — n фигур. Выделим из доски какой-нибудь прямоугольник 2*6. С одной стороны, этот прямоугольник можно разбить на три квадрата 2*2, и значит в нём 3m фигур. С другой стороны, его можно разрезать на четыре прямоугольника 1*3, и тогда в нём 4n фигур. Получаем соотношение 3m = 4n, откуда n делится на 3. Но n может принимать значения 0, 1, 2, 3. Таким образом, n = 0 или n = 3. Иными словами, либо все прямоугольники 1*3 пустые, и тогда на доске стоит 0 фигур, либо все прямоугольники 1*3 полностью заняты фигурами, и в этом случае на доске стоят 64 фигуры.",0.64,school,8 +"Мотоциклист Вася запланировал поездку из пункта А в пункт Б с постоянной скоростью. Первую половину пути он проехал со скоростью 𝑣1 — на 15 % меньшей, чем хотел. Затем он увеличил скорость до 𝑣2 и приехал в пункт Б точно в тот момент, в какой и планировал. Найдите 𝑣2/𝑣1","Пусть Вася изначально планировал ехать со скоростью 𝑣 в течение времени 𝑡. Тогда в текущей поездке первая половина пути составляет 0,5𝑡𝑣, и Вася проехал её со скоростью 0,85𝑣. Значит, времени он затратил на неё 0,5𝑣 / 0,85𝑣 = (1 / 1,7) * 𝑡 Тогда времени у него осталось 𝑡 - (1 / 1,7) * 𝑡 = (0,7 / 1,7) * 𝑡 Ему нужно проехать расстояние 0,5𝑡𝑣 с постоянной скоростью, поэтому ему нужно ехать со скоростью 0,5𝑡𝑣 / ((0,7 / 1,7) * 𝑡 ) = (0,5 * 1,7𝑣) / 0,7 = 0,85𝑣 / 0,7 . Осталось лишь найти ответ: 𝑣2 / 𝑣1 = (0,85𝑣 / 0,7) / 0,85𝑣 = 1 / 0,7=10 / 7",10/7,school,9 +"В пиццерии в каждую пиццу обязательно кладут помидоры и моцареллу. При заказе пиццы надо выбрать одну или несколько начинок: ветчину, грибы, салями или курицу. Также надо выбрать размер пиццы — 25, 30, 35 или 40 сантиметров. Сколько вариантов пиццы можно заказать в пиццерии? Пиццы считаются разными, если они имеют разные размеры или различаются хотя бы одним видом начинки","Сначала поймём, сколько существует вариантов положить начинку в пиццу. Каждый из 4 ингредиентов можно либо брать, либо не брать, итого получаем 2 * 2 * 2 * 2 = 16 вариантов. Но среди них есть один, когда мы ничего не кладём. По условию задачи такой вариант нам не подходит, а все остальные подходят, а значит, всего подходящих вариантов начинки 15. Для каждого размера пиццы возможен каждый из вариантов начинки, поэтому всего вариантов пиццы 15 * 4 = 60. +",60,school,9 +"Петя задумал составное натуральное число N, меньшее 1000.Он выписал на доску все натуральные делители N, не равные 1. Оказалось, что два наименьших числа на доске различаются на 39. Чему может быть равно N? Укажите все возможные варианты.","Предположим, что 𝑁 нечётно. Тогда все делители 𝑁 тоже нечётны, а значит, разность между любыми двумя чётная, и поэтому не может равняться 39. Таким образом, 𝑁 точно чётно. Отсюда следует, что наименьшим числом на доске будет 2, тогда следующим по величине числом будет 2 + 39 = 41. Поскольку 𝑁 делится на два взаимно простых числа 2 и 41, то оно делится и на их произведение 82. Легко убедиться, что число 𝑁 = 82 подходит под условие задачи. Предположим, что существует ещё какое-то подходящее 𝑁 != 82. Поскольку двумя его наименьшими делителями, отличными от 1, являются 2 и 41, то у 𝑁 нет других делителей в интервале (2; 41). Тогда 𝑁 должно делиться либо на какое-то простое число, большее 41, либо на степени чисел 2 или 41. При этом делиться на какую-то ещё степень 2 число 𝑁 не может, т.к. среди делителей 𝑁 не должно быть числа 4. Если же 𝑁 делится на простое число, большее 41, или на 412 , то 𝑁 не меньше 2 * 41 * 41 = 3362 > 1000. Таким образом, единственный возможный вариант для 𝑁 — это 82.",82,school,9 +"На острове живут рыцари, которые всегда говорят правду, и лжецы, которые всегда лгут. Однажды собралось несколько жителей острова, и каждый из них произнёс по одной фразе: Один сказал: «Среди нас не более 9 рыцарей». Двое сказали: «Среди нас не более 8 рыцарей». Трое сказали: «Среди нас не более 7 рыцарей». ... Девять человек сказали: «Среди нас не более 1 рыцаря». А все остальные сказали: «Среди нас не более 10 рыцарей». Сколько человек могло сказать последнюю фразу? Укажите все возможные варианты.","Рассмотрим случаи того, сколько всего рыцарей среди собравшихся. Случай 1. Рыцарей хотя бы 11. В этом случае все фразы оказываются ложью. А поскольку каждый человек что-то сказал, то такой случай нев��зможен. Случай 2. Рыцарей ровно 10. Тогда правду говорят только говорящие последнюю фразу (и они рыцари), а все остальные говорят неправду (и они лжецы). Значит, последнюю фразу говорят все рыцари, т. е. 10 человек. Случай 3. Рыцарей ровно 9. Тогда правду говорит человек, сказавший первую фразу, и люди, говорящие последнюю фразу (а все остальные говорят неправду, и поэтому являются лжецами). Значит, последнюю фразу говорит 9 − 1 = 8 человек. Случай 4. Рыцарей ровно 8. Тогда правду говорят только люди, сказавшие первую, вторую и последнюю фразы. Значит, последнюю фразу говорит 8 − 1 − 2 = 5 человек. Случай 5. Рыцарей ровно 7. Тогда правду говорят только люди, сказавшие первую, вторую, третью и последнюю фразы. Значит, последнюю фразу говорит 7 − 1 − 2 − 3 = 1 человек. Случай 6. Рыцарей не более 6. Тогда правду говорят как минимум люди, сказавшие первую, вторую, третью и четвёртую фразы, т. е. рыцарей не менее +1 + 2 + 3 + 4 = 10 человек. Противоречие. Значит, этот случай невозможен. Таким образом, количество людей, сказавших последнюю фразу, равно 10, 8, 5 или 1. Ясно, что все эти варианты возможны.",15810,school,9 +"В магазине продаётся 20 товаров, стоимости которых — различные натуральные числа от 1 до 20 рублей. Магазин решил устроить акцию: при покупке любых 5 товаров один из них выдаётся в подарок, причём покупатель сам выбирает, какой товар получит бесплатно. Влад хочет купить все 20 товаров в этом магазине, заплатив как можно меньше. Сколько рублей ему понадобится? (Каждый из 20 товаров продаётся в 1 экземпляре.)","Влад может воспользоваться акцией не более 4 раз, поэтому он бесплатно приобретёт не более 4 товаров. Суммарная стоимость этих 4 товаров не превосходит 17 + 18 + 19 + 20 рублей. Значит, рублей Владу надо не менее (1 + 2 + 3 + … + 20) − (17 + 18 + 19 + 20) = 1 + 2 + 3 + … + 16 = 16 * 17 / 2 = 136. Покажем, что 136 рублей ему точно хватит. Он может совершать покупки товаров со следующими стоимостями: (1, 2, 3, 4, 17), (5, 6, 7, 8, 18), (9, 10, 11, 12, 19), (13, 14, 15, 16, 20). Если Влад в каждой покупке будет брать последний товар бесплатно, то потратит в точности 136 рублей.",136,school,9 +"Ваня загадал два натуральных числа, произведение которых равняется 7200. Какое наибольшее значение может принимать НОД этих чисел?","Поскольку каждое из этих чисел делится на их НОД, то их произведение делится на квадрат этого НОД. Наибольший точный квадрат, на который делится число 7200 = (2**5) * (3**2) * (5**2) — это 3600 = (2**2 * 3 * 5)**2 , поэтому НОД двух искомых чисел не превосходит 60. При этом НОД может равняться 60, если искомые два числа — это 60 и 120",60,school,9 +"Простое число 𝑝 таково, что число 𝑝 + 25 является седьмой степенью простого числа. Чему может быть равно 𝑝? Укажите все возможные варианты.","Воспользуемся тем, что единственное простое чётное число — это 2. Пусть 𝑝 = 2, тогда 𝑝 + 25 = 27, что не является седьмой степенью. Противоречие. Пусть 𝑝 > 2, тогда 𝑝 нечётно, а 𝑝 + 25 чётно. Поскольку 𝑝 + 25 чётно и является седьмой степе��ью простого числа, то это простое число равно 2. Следовательно, 𝑝 + 25 = 27 = 128, откуда получаем 𝑝 = 103.",103,school,9 +"На острове живут рыцари, которые всегда говорят правду, и лжецы, которые всегда лгут. Однажды собрались 80 жителей острова, все они надели на себя футболки с номерами от 1 до 80 (у разных жителей разные номера). Каждый из них сказал одну из двух фраз: «Среди собравшихся хотя бы у 5 лжецов номер футболки больше моего». «Среди собравшихся хотя бы у 5 лжецов номер футболки меньше моего». Какое наименьшее количество рыцарей могло быть среди этих 80 жителей?","Предположим, что лжецов хотя бы 11. Упорядочим по возрастанию номера на их футболках и выберем лжеца с 6-м по счёту номером. Тогда он обязан сказать правду, ведь есть хотя бы 5 лжецов как с меньшим номером, так и с бо́льшим. Таким образом, лжецов не более 10, т. е. рыцарей не менее 70. Покажем, что рыцарей могло быть ровно 70. Пусть, например, рыцари были в футболках с номерами 1—70, а лжецы — в футболках с номерами 71—80. Все рыцари и лжецы с номерами 76—80 сказали первую фразу, а лжецы с номерами 71—75 сказали вторую фразу. Ясно, что все условия задачи выполняются",70,school,9 +"Дан квадратный трёхчлен 𝑃(𝑥), старший коэффициент которого равен 1. На графике 𝑦 = 𝑃(𝑥) отметили две точки с абсциссами 10 и 30. Оказалось, что биссектриса первой четверти координатной плоскости пересекает отрезок между ними в его середине. Найдите 𝑃(20).","Середина этого отрезка имеет координаты ((10 + 30)/2, (𝑃(10) + 𝑃(30))/2 ). Поскольку она лежит на биссектрисе первой четверти, т.е. на прямой 𝑦 = 𝑥, эти координаты равны. Отсюда получаем 𝑃(10) + 𝑃(30) = 40. Так как 𝑃(𝑥) приведённый, его можно записать в виде 𝑃(𝑥) = 𝑥**2 + 𝑎𝑥 + 𝑏. Тогда условие 𝑃(10) + 𝑃(30) = 40 переписывается в виде 100 +10𝑎 + 𝑏 +900 +30𝑎 + 𝑏 = 40, откуда следует, что 40𝑎 + 2𝑏 = −960 и 20𝑎 + 𝑏 = −480. Следовательно, 𝑃(20) = 400 + 20𝑎 + 𝑏 = 400 - 480 = -80.",-80,school,9 +"На острове живут красные, жёлтые, зелёные и синие хамелеоны. В пасмурный день либо один красный хамелеон меняет окрас на жёлтый цвет, либо один зелёный хамелеон — на синий цвет. В солнечный день либо один красный хамелеон меняет окрас на зелёный цвет, либо один жёлтый хамелеон — на синий цвет. В сентябре было 18 солнечных и 12 пасмурных дней. При этом количество жёлтых хамелеонов увеличилось на 5. На сколько увеличилось количество зелёных хамелеонов?","Пусть 𝐴 — количество зелёных хамелеонов на острове, а 𝐵 — жёлтых. Рассмотрим величину 𝐴 − 𝐵. Заметим, что каждый пасмурный день она уменьшается на 1, а каждый солнечный день она увеличивается на 1. Поскольку солнечных дней в сентябре было на 18 − 12 = 6 больше, чем пасмурных, то и величина 𝐴−𝐵 увеличилась на 6 за это время. Поскольку 𝐵 увеличилось на 5, то 𝐴 должно было увеличиться на 5 + 6 = 11",11,school,9 +"У Дениса есть карточки с числами от 1 до 50. Сколько существует способов выбрать две карточки так, чтобы разность чисел на карточках равнялась 11, а произведение делилось на 5? Порядок выбранных карточек не важен: например, способ выбора карточек с числами 5 +и 16, а также способ выбора карточек с числами 16 и 5 — это один и тот же способ. +","Чтобы произведение делилось на 5, необходимо и достаточно, чтобы какой-то множитель делился на 5. Пусть 𝑛 — делящееся на 5 выбранное число, тогда в пару ему надо выбрать 𝑛 − 11 или 𝑛 + 11, притом оба выбранных числа должны быть натуральными. Ясно, что при 𝑛 = 5, 10, 40, 45, 50 есть только один способ выбрать пару, а при 𝑛 = 15, 20, 25, 30, 35 есть ровно два способа выбрать пару. Итого 5 * 1 + 5 * 2 = 15 способов.",15,school,9 +"Торговцы Андрей и Борис купили по 60 мешков картошки у одного и того же фермера. Все мешки стоили одинаково. Андрей продал все свои мешки, увеличив их цену на 100%. Борис же сначала увеличил цену на 60%, а когда продал 15 мешков, увеличил цену ещё на 40% и продал остальные 45 мешков. Оказалось, что Борис заработал на 1200 рублей больше Андрея. Сколько рублей стоил один мешок картошки у фермера?","Пусть мешок у фермера стоил 𝑥 рублей. На покупку 60 мешков Андрей и Борис потратили поровну. Из условия следует, что Андрей продал 60 мешков по 2𝑥 рублей, т. е. получил 60 * 2𝑥 рублей. Борис же продал 15 мешков по цене 1,6𝑥 рублей, а затем продал 45 мешков по цене 1,6𝑥 * 1,4 рублей. Получаем уравнение 2𝑥 * 60 + 1200 = 15 * 1,6𝑥 + 45 * 1,6𝑥 * 1,4. Решая его, находим 𝑥 = 250.",250,school,9 +"Антон, Вася, Саша и Дима ехали на машине из города А в город Б, каждый из них по очереди был за рулём. Весь путь машина ехала с постоянной скоростью. Антон вёл машину в два раза меньше, чем Вася, а Саша вёл машину столько же, сколько Антон и Дима вместе взятые. Дима был за рулём лишь десятую часть пути. Какую часть пути за рулём был Вася? Ответ запишите в виде десятичной дроби.","Пусть доля пути, в течение которой Антон был за рулём, равна 𝑥. Тогда доля Васи равна 2𝑥, Димы — 0,1, а Саши — 0,1 + 𝑥. Суммируя все доли, получаем 4𝑥 + 0,2, что должно составить 1. Отсюда 𝑥 = 0,2; а ответом будет 2𝑥 = 0,4.",0.4,school,9 +"У Буратино есть много монет по 5 и по 6 сольдо, каждого вида более 10 монет. Придя в магазин и купив книгу за 𝑁 сольдо, он понял, что не сможет за неё рассчитаться без сдачи. Какое наибольшее значение может принимать натуральное 𝑁, если оно не больше 50?","Легко проверить, что при 𝑁 = 19 сдача необходима. Заметим, что числа от 20 до 24 не подходят под условие, ведь за них можно было бы рассчитаться без сдачи: 𝑁 = 20 = 4 * 5, 21 = 3 * 5 + 6, 22 = 2 * 5 + 2 * 6, 23 = 5 + 3 * 6, 24 = 4 * 6. Ясно, что тогда и числа от 25 до 50 не подходят под условие, ведь каждое из них является суммой одного или нескольких слагаемых, равных 5, а также числа от 20 до 24, являющегося суммой слагаемых, каждое из которых равно 5 и 6.",19,school,9 +"У уравнений x**2 + 2019ax + b = 0 и x**2 + 2019bx + a = 0 есть один общий корень. Чему может быть равен этот корень, если известно, что a != b?","Пусть общий корень данных уравнений равен r. Тогда r**2 + 2019ar + b = 0 = r**2 + 2019br + a. Отсюда получаем, что 2019r(a − b) = (a − b). Поскольку a != b, из этого следует, что r = 1/2019 .",1/2019,school,9 +Есть три брата-акробата. Их средний рост — 1 метр 74 сантиметра. А средний рост двух из этих братьев: самого высокого и самого низкого — 1 метр 75 сантиметров. Какого роста средний брат? Ответ обоснуйте.,"Поскольку средний рост всех трёх — 1 метр 74 сантиметра, суммарный рост всех составляет 5 метров 22 сантиметра. Средний рост двух братьев равен 1 метр 75 сантиметров, поэтому их суммарный рост составляет 3 метра 50 сантиметров. А значит, рост среднего брата составляет 1 метр 72 сантиметра.",1.72,school,9 +"Назовём трёхзначное число интересным, если хотя бы одна его цифра делится на 3. Какое наибольшее количество подряд идущих интересных чисел может быть? (Приведите пример и докажите, что больше чисел получить нельзя.)","Числа 289, 290, . . . , 299, 300, . . . , 399, 400, . . . , 409, 410 являются интересными (напомним, что 0 делится на 3), и их всего 122. Докажем, что большего количества быть не может. Предположим, что нам удалось найти большее количество подряд идущих интересных чисел; выберем из них 123 подряд идущих. Назовём сотню подряд идущих чисел, у которых разряд сотен одинаков и делится на 3, интересной сотней. Заметим, что до любой интересной сотни идут только 11 интересных чисел, оканчивающихся на 89, 90, . . . , 99, а 12-е число оканчивается на 88 и интересным не будет. Аналогично после интересной сотни идут тоже только 11 интересных чисел, оканчивающихся на 00, . . . , 09, 10, а 12-е число оканчивается на 11 и также не интересное. Если наша последовательность из 123 чисел пересекается с некоторой интересной сотней, то она содержит хотя бы 12 чисел либо до, либо после этой сотни. Следовательно, хотя бы одно число в ней не интересное. Если же наша последовательность из 123 чисел не пересекается с интересной сотней, то она содержит хотя бы одно число, оканчивающееся на 55 (как и на любую другую комбинацию цифр). Но это число не интересное, так как ни один разряд в нём на 3 не делится",122,school,9 +"45 конфет стоят столько же рублей, сколько их можно купить на 20 рублей. Сколько конфет можно купить на 50 рублей?","Пусть x — стоимость одной конфеты в рублях. Тогда 45x = 20 / x , откуда x = 2 / 3 . Тогда на 50 рублей можно купить 50 / x = 75 конфет.",75,school,9 +"Чебурашка и Гена съели торт. Чебурашка ел вдвое медленнее Гены, но начал есть на минуту раньше. В итоге им досталось торта поровну. За какое время Чебурашка съел бы торт в одиночку?","Первый способ. Если Чебурашка ест вдвое медленнее Гены, то, чтобы съесть столько же торта, сколько съел Гена, ему нужно в два раза больше времени. Значит, то время, которое Чебурашка ел в одиночку (1 минута), составляет половину всего времени, за которое Чебурашка съел половину торта. Таким образом половину торта он съел за 2 минуты, а весь торт съел бы за 4 минуты. Второй способ. Пусть Гена съедает весь торт за x минут, тогда Чебурашке на весь торт нужно 2x минут. Каждому из них доста��ась половина торта, то есть Гена ел 0,5x минут, а Чебурашка x минут. Из условия следует, что 0,5x + 1 = x, откуда x = 2. Значит, Чебурашка съест торт за 2 * 2 = 4 минуты.",4,school,9 +"На доске была написана несократимая дробь. Петя уменьшил её числитель на 1, а знаменатель на 2. А Вася прибавил к числителю 1, а знаменатель оставил без изменений. Оказалось, что в результате мальчики получили одинаковые значения. Какой именно результат у них мог получиться?","Пусть была написана дробь a / b . Тогда Петя получил (a - 1) / (b - 2), а Вася (a + 1) / b. Так как они получили одинаковый результат (a - 1) / (b - 2) = (a + 1) / b, откуда b - a = 1. Значит, исходная дробь имела вид a / (a + 1). И Петя получил из неё дробь (a - 1) / (a - 1), а Вася (a + 1) / (a + 1), т. е. результат и Пети, и Васи равен 1.",1,school,9 +"Дима должен был попасть на станцию в 18:00. К этому времени за ним должен был приехать отец на автомобиле. Однако Дима успел на более раннюю электричку и оказался на станции в 17:05. Он не стал дожидаться отца и пошёл ему навстречу. По дороге они встретились, Дима сел в автомобиль, и они приехали домой на 10 минут раньше рассчитанного времени. С какой скоростью шёл Дима до встречи с отцом, если скорость автомобиля была 60 км/ч?","Дима приехал домой на 10 минут раньше, за это время автомобиль дважды проехал бы путь, который Дима прошёл. Следовательно, на пути к вокзалу отец на автомобиле сэкономил 5 минут и встретил Диму в 17:55. Значит, Дима прошёл расстояние от вокзала до встречи с отцом за 50 минут, то есть он шёл в 10 раз медленнее автомобиля, и его скорость была 6 км/ч",6,school,9 +"В подземном царстве живут гномы, предпочитающие носить либо зелёные, либо синие, либо красные кафтаны. Некоторые из них всегда лгут, а остальные всегда говорят правду. Однажды каждому из них задали четыре вопроса. 1. «Ты предпочитаешь носить зелёный кафтан?» 2. «Ты предпочитаешь носить синий кафтан?» 3. «Ты предпочитаешь носить красный кафтан?» 4. «На предыдущие вопросы ты отвечал честно?» На первый вопрос «да» ответили 40 гномов, на второй — 50, на третий — 70, а на четвёртый — 100. Сколько честных гномов в подземном царстве?","На 4-й вопрос и честный, и лгун ответят «да», поэтому в подземном царстве всего 100 гномов. Честный гном на один из трёх первых вопросов ответит «да», а на два — «нет». А лгун, наоборот, на два из первых трёх вопросов ответит «да», а на один — «нет». Пусть всего x честных гномов. Тогда всего на первые три вопроса +будет x + 2 * (100 – x) = 200 – x ответов «да», т. е. 200 – x = 40 + 50 + 70 = 160, +откуда x = 40. ",40,school,9 +"На доске записано 7 различных чисел, сумма которых равна 10. Петя умножил каждое из них на сумму остальных шести и записал 7 полученных произведений в тетрадь. Оказалось, что в тетради встречаются только четыре различных числа. Найдите одно из чисел, записанных на доске","Для каждого числа x, написанного на доске, произведение x и суммы шести оставшихся равно f(x) = x * (10−x) = 10x − x**2 . Квадратичная функция f(x) принимает все значения, кроме максимального, два раза — а именно, в точках a и 10 − a. Значит, если f(a) = f(b) при a != b, то a + b = 10. Таким образом, каждое число встречается в тетради не более двух раз. Значит, так как в тетради всего четыре различных числа, три из них встречаются по два раза, и ещ¨е одно — один раз. Таким образом, шесть из семи чисел на доске разбиваются на пары так, что сумма чисел каждой пары равна 10. Значит, сумма этих шести чисел равна 30, тогда седьмое число равно 10 − 30 = −20",-20,region,9 +"На окружности длиной 1 метр отмечена точка. Из нее в одну и ту же сторону одновременно побежали два таракана с различными постоянными скоростями. Каждый раз, когда быстрый таракан догонял медленного, медленный мгновенно разворачивался, не меняя скорости. Каждый раз, когда они встречались лицом к лицу, быстрый мгновенно разворачивался, не меняя скорости. На каком расстоянии от отмеченной точки могла произойти их сотая встреча?","Назовем быстрого и медленного таракана B и M соответственно. Если таракан бежит в том же направлении, что и в момент старта, то будем говорить, что он бежит вперед, в противном случае будем говорить, что он бежит назад. До первой встречи оба таракана бегут вперед, между первой и второй встречами B бежит вперед, а M — назад. Между второй и третьей встречами оба таракана бегут назад, а между третьей и четвертой встречами B бежит назад, а M — вперед. Наконец, на четвертой встрече B разворачивается, и они оба снова начинают бег вперед. Будем следить за перемещением M. Если между двумя встречами тараканы бегут в противоположные стороны, между такими встречаем всегда проходит одно и то же время, а значит, M всегда пробегает одно и то же расстояние. Таким образом, между первой и второй встречами, а также между третьей и четвертой встречами M пробегает одно и то же расстояние в противоположных направлениях. Аналогично, когда между двумя встречами тараканы бегут в одном направлении, это тоже всегда занимает одинаковое время, и M пробегает одно и то же расстояние. Таким образом, до первой встречи, а также между второй и третьей встречами M также пробегает одно и то же расстояние в противоположных направлениях. Стало быть, в момент четвертой встречи M (а значит, и B) будет в точке старта. Далее эта ситуация будет повторяться каждые 4 встречи. Следовательно, в точке старта тараканы будут и в момент сотой встречи.",0,region,9 +"Правильный треугольник T со стороной 111 разбит прямыми, параллельными его сторонам, на правильные треугольники со стороной 1. Все вершины этих треугольников, кроме центра треугольника T, отмечены. Назов¨ем множество из нескольких отмеченных точек линейным, если все эти точки лежат на одной прямой, параллельной стороне T. Сколько существует способов разбить все отмеченные точки на 111 линейных множеств? (Способы, отличающиеся порядком множеств, считаются одинаковыми.)","Рассмотрим равносторонний треугольник со стороной k, разобьем его на правильные треугольнички со стороной 1 и отметим все вершины этих треугольничков; полученную конструкцию назовем k-треугольником. В дальнейшем под прямыми мы всегда будем понимать прямые, параллельные сторонам этого треугольника и проходящие через хотя бы одну отмеченную точку. +Лемма. Пусть A — отмеченная точка в k-треугольнике. Тогда существует единственный способ провести k прямых +так, что все отмеченные точки, кроме, возможно, A, покрыты этими прямыми. А именно, для каждой стороны k треугольника надо провести все прямые, параллельные ей и лежащие между этой стороной и точкой A (включая саму сторону, но исключая прямую, содержащую A). +Доказательство. Индукция по k. База при k = 1 проверяется легко: надо провести прямую, содержащую две оставшихся точки, кроме A. Для перехода рассмотрим сторону k-треугольника, на которой не лежит A. Если прямая, содержащая эту сторону, не проведена, то все k + 1 отмеченных точек на этой прямой должны +быть покрыты различными прямыми; это невозможно, так как прямых k. Значит, эта прямая проведена. Выкинув ее и точки k-треугольника, лежащие на ней, получаем (k −1) - треугольник, в котором проведено k − 1 прямых с теми же условиями. Осталось применить предположение индукции Перейдем к задаче. Рассмотрим одно из разбиений на линейные множества. Для каждого множества проведем прямую, его содержащую. Тогда эти прямые покрыли все отмеченные точки 111-треугольника, кроме, возможно, его центра A. Значит, эти прямые устроены так, как описано в лемме, и для любого разбиения этот набор прямых один и тот же. Заметим, что наш 111-треугольник разбился на 6 областей: три «ромба» в углах, состоящих из точек, покрытых нашими прямыми дважды, и три «трапеции» у сторон, в которых каждая точка покрыты одной прямой. Тогда каждая точка в «трапеции» относится к множеству, лежащему на этой прямой; каждую же точку в «ромбе» можно отнести к любому из двух множеств, лежащих на проходящих через нее прямых. Все такие выборы можно сделать независимо друг от друга. Поскольку в каждом из трех «ромбов» всего 37**2 точек, получаем, что требуемых разбиений ровно 2**(3 * (37**2)) = 2**4107",2**4107,region,9 +"На доску записали 99 чисел, среди которых нет равных. В тетрадку выписали (99 * 98) / 2 чисел — все разности двух чисел с доски (каждый раз из большего числа вычитали меньшее). Оказалось, что в тетрадке число 1 записано ровно 85 раз. Пусть d — наибольшее число, записанное в тетрадке. Найдите наименьшее возможное значение d.","Докажем, что d >= 7. Все числа с доски разбиваются на цепочки чисел вида a, a + 1, a + 2, . . . , a + t так, что числа из разных цепочек не отличаются ровно на 1. Такое разбиение нетрудно построить, соединив любые два числа, отличающиеся на 1, отрезком и рассмотрев полученные ломаные. Пусть получилось k цепочек, в которых n1, n2, . . . , nk чисел соответственно (некоторые цепочки могут состоять из одного числа). В цепочке из ni чисел есть ровно ni − 1 пара чисел, отличающихся на 1. Поэтому общее количество единиц в тетрадке равно (n1 − 1) + (n2 − 1) +. . .+ (nk − 1) = (n1 + n2 +. . .+ nk) − k = 99 − k, +откуда k = 99 − 85 = 14. Значит, в одной из цепочек не меньше, чем 99 / 14 чисел, то есть не меньше 8 чисел. Разность наибольшего и наименьшего чисел в такой цепочке не меньше 8 − 1 = 7. Осталось привести пример, в котором d = 7. Такой пример дают, например, числа 0 = 0 / 14, 1 / 14, 2 / 14, ..., 98 / 14 = 7 Действительно, в этом примере d = 7, и ровно для первых 85 из этих чисел в наборе есть число, на единицу большее.",7,region,9 +На доске девять раз (друг под другом) написали некоторое натуральное число N. Петя к каждому из 9 чисел приписал слева или справа одну ненулевую цифру; при этом все приписанные цифры различны. Какое наибольшее количество простых чисел могло оказаться среди 9 полученных чисел?,"Пусть S — сумма цифр числа N. Тогда суммы цифр полученных чисел будут равны S + 1, S + 2, . . . , S + 9. Три из этих сумм будут делиться на 3. По признаку делимости на 3, соответствующие три числа на доске также будут делиться на 3. При этом они будут больше 3, а значит, будут составными. Поэтому больше 6 простых чисел на доске оказаться не может. Шесть простых чисел может оказаться даже при N = 1 — например, если Петя получит, среди прочих, числа 11, 13, 41, 61, 17 и 19.",6,region,9 +"В компании некоторые пары людей дружат (если A дружит с B, то и B дружит с A). Оказалось, что среди каждых 100 человек в компании количество пар дружащих людей неч¨етно. Найдите наибольшее возможное количество человек в такой компании","Во всех решениях ниже мы рассматриваем граф дружб, в котором вершины — это люди в компании, а два человека соединены ребром, если они дружат. Если граф — это цикл, содержащий 101 вершину, то на любых 100 вершинах ровно 99 ребер, так что такая компания удовлетворяет условиям задачи. Осталось показать, что не существует такой компании из 102 человек (тогда и компании из более чем 102 человек тоже быть не может). Существует всего n = 51 * 101 способов выбросить две вершины из 102, оставив 100. Пронумеруем эти способы числами от 1 до n. Пусть ai — количество ребер на оставшихся 100 вершинах в i-м способе; по предположению, все числа ai нечетны, а значит, нечетна и их сумма S (поскольку число n нечетно). С другой стороны, рассмотрим любое ребро uv. Это ребро учтено в числе ai ровно тогда, когда вершины u и v не выброшены в i-м способе, то есть когда выброшена какая-то пара из оставшихся 100 вершин. Это происходит в k = 50 * 99 способах. Итак, каждое ребро учтено в S четное количество k раз, поэтому S должно быть четным. Противоречие.",101,region,9 +"Десятизначные натуральные числа a, b, c таковы, что a + b = c. Какое наибольшее количество из 30 их цифр могут оказаться нечетными?","Заметим, что если a + b = c, то все три числа a, b, c не могут оказаться одновременно нечетными. Следовательно, среди них есть как минимум одно четное число, и последняя цифра этого числа также будет четной. Таким образом, среди 30 цифр есть как минимум одна четная, а нечетных — не более 29. Пример 1 999 999 999 + 1 111 111 111 = 3 111 111 110, показывает, что среди 30 цифр могут оказаться ровно 29 нечетных.",29,region,9 +"Если на столе лежит несколько кучек камней, считается, что на столе много камней, если можно найти 50 кучек и пронумеровать их числами от 1 до 50 так, что в первой кучке есть хотя бы один камень, во второй — хотя бы два камня, . . . , в пятидесятой — хотя бы пятьдесят камней. Пусть исходно на столе лежат 100 кучек по 100 камней в каждой. Найдите наибольшее n <= 10 000 такое, что после удаления из исходных кучек любых n камней на столе вс¨е равно останется много камней. (При удалении камней кучка не распадается на несколько.)","Если удалить полностью 51 кучку, то, очевидно, не останется много камней. Значит, искомое значение n меньше 5100. (Альтернативно, можно удалить из всех кучек по 51 камню.) Осталось показать, что при удалении любых n = 5099 камней останется много камней. Пусть в кучках осталось a1, a2, . . . , a100 камней соответственно; можно считать, что 0 <= a1 <= a2 <= ... <= a100 <= 100. Покажем, что ai+50 > i при i = 1, 2, . . . , 50, то есть кучки с номерами от 51 до 100 удовлетворяют требованиям. Пусть это не так, то есть ai+50 <= i−1. при некотором i <= 50. Это значит, что каждая из первых i + 50 кучек содержит не более i−1 камня, то есть из нее удалено хотя бы 101−i камней. Поэтому общее количество удаленных камней не меньше, чем (i + 50)(101 − i) = 5100 − (i − 1)(i − 50) >= 5100. Противоречие.",5099,allrus,9 +"Для какого наименьшего натурального числа a существуют целые числа b и c такие, что квадратный тр¨ехчлен ax**2 + bx + c имеет два различных положительных корня, не превосходящих 1/1000?","Положим для краткости n = 1000. Пусть x1 и x2 — два различных корня трехчлена f(x) = ax**2 + bx + c, причем 0 < x1 < x2 <= 1 n . Тогда число b = −a(x1 + x2) отрицательно, а число c = ax1x2 положительно. Более того, имеем −b/a = x1 + x2 < 2/n , откуда a > − nb/2 . Поскольку корни различны, дискриминант D = b**2 − 4ac положителен. Следовательно, b**2 > 4ac > −2nbc и, значит, −b > 2nc. Поэтому a > (−b) * (n/2) > 2nc * (n/2) = (n**2)c. Пусть a = (n**2)c + d, где d — натуральное число. Предположим, что a < n**2 + n. Тогда c = 1 и d < n. Стало быть, 0 <= f(1/n) = a/(n**2) + b/n + c = d/(n**2) + b/n +2 < 1/n + b/n + 2 и, значит, −b < 2n + 1. Следовательно, −b <= 2n и D = b**2 − 4ac <= 4n**2- 4(n**2 + d) = -4d < 0. Это противоречие показывает, что d >= n. Если же a = n**2 + n, то при b = −2n − 1 и c = 1 тр¨ехчлен имеет корни x1 = 1/(n + 1) и x2 = 1/n",1001000,allrus,9 +"На окружности красным цветом записали четыре различных натуральных числа. На дуге между каждыми двумя соседними красными числами записали синим цветом их произведение. Известно, что сумма всех четырёх синих чисел равна 1133. Найдите сумму всех красных чисел.","Обозначим исходные числа через 𝑎, 𝑏, 𝑐, 𝑑 в порядке по часовой стрелке. Тогда синим цветом записаны числа 𝑎𝑏, 𝑏𝑐, 𝑐𝑑, 𝑑𝑎. Сумма этих чисел равна 𝑎𝑏 + 𝑏𝑐 + 𝑐𝑑 + 𝑑𝑎 = (𝑎 + 𝑐)(𝑏 + 𝑑) = 1133. ��аметим, что числа 𝑎 + 𝑐 и 𝑏 + 𝑑 больше 1, поскольку являются суммой двух натуральных чисел. Но число 1133 единственным образом раскладывается в произведение двух чисел, больших единицы: 1133 = 11 * 103. Значит, 𝑎 + 𝑐 = 11, 𝑏 + 𝑑 = 103 или наоборот. В любом случае, 𝑎 + 𝑏 + 𝑐 + 𝑑 = 11 + 103 = 114.",114,school,10 +"В клетках таблицы 11 * 11 расставили числа от 1 до 121, каждое по разу. В каждой строке все числа идут по возрастанию слева направо, и в каждом столбце все числа идут по возрастанию сверху вниз. Назовём число особым, если оно отличается от каждого своего соседа хотя бы на 2. Какое наибольшее количество особых чисел может быть? Числа являются соседями, если они стоят в соседних по стороне клетках.","Число 1 меньше всех остальных чисел, поэтому оно должно быть первым в строке и первым в столбце, а значит, оно должно находиться в левом верхнем углу. Значит, число 2 не может быть в левом верхнем углу, поэтому либо оно не в первой строке, либо не в первом столбце. В первом случае оно должно быть больше какого-то числа в его столбце, а во втором случае — какого-то числа в его строке. Но единственное число, которое меньше 2 — это 1, поэтому числа 1 и 2 должны находиться рядом. Тогда никакое из них не может быть особым. Аналогичными рассуждениями легко получить, что число 121 должно находиться в правом нижнем углу, а число 120 должно быть рядом с ним, поэтому эти числа также не могут быть особыми. Значит, особых чисел не более 121 − 4 = 117. ",117,school,10 +"Саша уже неделю смотрит все серии интересного сериала подряд. Вчера Саша посмотрел 9 серий, а сегодня всего 6. Оказалось, что сумма номеров всех серий, просмотренных вчера, равна сумме номеров всех серий, просмотренных сегодня. Какой номер имеет последняя просмотренная Сашей серия? (Серии нумеруются последовательными натуральными числами, начиная с 1.)","Обозначим за 𝑥 номер первой серии, которую Саша посмотрел вчера. Тогда номера остальных серий, просмотренных вчера, — это 𝑥 + 1, 𝑥 + 2, … , 𝑥 + 8, и сумма номеров всех вчерашних серий равна 9𝑥 + (1 + 2 + … + 8) = 9𝑥 + 36. Сегодня Саша посмотрел серии с номерами 𝑥 + 9, 𝑥 + 10, … , 𝑥 + 14, сумма которых равна 6𝑥 + (9 + 10 + 11 + 12 + 13 + 14) = 6𝑥 + 69. Получаем уравнение 9𝑥 + 36 = 6𝑥 + 69, откуда 𝑥 = 11. Таким образом, последняя серия, просмотренная сегодня, имеет номер 𝑥 + 14 = 25.",25,school,10 +"В магазине продаются 15 видов шоколада. За неделю Андрей попробовал 6 видов, Борис — 11, а Денис — 13. Оказалось, что ни один вид шоколада не продегустировали все трое. Сколько видов шоколада попробовали и Андрей, и Борис?","Андрей не попробовал 9 видов шоколада, Борис — 4, Денис — 2. В сумме это 15, и поскольку каждый из 15 видов хотя бы кто-то не попробовал, то эти не попробованные ребятами виды шоколада не должны пересекаться. Следовательно, Андрей и Борис не попробовали в совокупности ровно 9 + 4 = 13 видов, т. е. попробовали вместе ровно 15 − 13 = 2 вида.",2,school,10 +Сумма трёх различных натуральных ��елителей нечётного натурального числа 𝑛 равна 10327. Какое наименьшее значение может принимать 𝑛?,"Наибольший делитель числа 𝑛 равен 𝑛. Поскольку это число нечётно, второй по величине его делитель не превосходит 𝑛/3 , а третий — не превосходит 𝑛/5 . Следовательно, сумма трёх любых различных делителей не превосходит 𝑛 + 𝑛/3 + 𝑛/5 = 23𝑛/15 . Получаем, что 10327 <= 23𝑛/15 , откуда 𝑛 >= 6735. Отметим также, что число 6735 подходит под условие (оно нечётно, делится на 3 и на 5): сумма трёх его наибольших делителей равна 6735 + 6735/3 + 6735/5 = 10327.",6735,school,10 +"Сколько существует троек натуральных чисел (𝑎, 𝑏, 𝑐), удовлетворяющих равенству max(𝑎, 𝑏) * max(𝑐, 12) = min(𝑎, 𝑐) * min(𝑏, 24)? Здесь min(𝑥, 𝑦) — это наименьшее из чисел 𝑥 и 𝑦, а max(𝑥, 𝑦) — наибольшее из чисел 𝑥 и 𝑦 ","Заметим, что max(𝑎, 𝑏) >= 𝑏 >= min(𝑏, 24), а max(𝑐, 12) >= 𝑐 >= min(𝑎, 𝑐). Поскольку все числа натуральные, равенство в условии достигается только тогда, когда все неравенства обращаются в равенства: max(𝑎, 𝑏) = 𝑏 = min(𝑏, 24) и max(𝑐, 12) = 𝑐 = min(𝑎, 𝑐). Это равносильно тому, что 12 <= 𝑐 <= 𝑎 <= 𝑏 <= 24. Значит, в реальности нужно найти количество упорядоченных троек натуральных чисел в отрезке [12; 24] (кстати, он содержит всего 13 натуральных чисел). По набору чисел, лежащих в этом отрезке, однозначно восстанавливается, чему равны 𝑎, 𝑏, 𝑐 (ведь 𝑐 <= 𝑎 <= 𝑏). Какие бывают варианты? 1) Все три числа разные. Количество способов выбрать 3 различных числа из 13 равно 286. 2) Какие-то два числа совпадают. Количество способов выбрать 2 различных числа равно 78. При этом нужно ещё установить, какое именно из этих чисел встречается дважды, поэтому количество способов в этом случае равно 78 * 2 = 156. 3) Все три числа совпадают. Количество способов в этом случае — 13. Итак, общее количество способов равно 286 + 156 + 13 = 455.",455,school,10 +"В каждую клетку таблицы 5 * 5 невидимыми чернилами вписано натуральное число. Известно, что сумма всех чисел равна 200, а сумма трёх чисел, находящихся внутри любого прямоугольника 1 * 3, равна 23. Чему равно центральное число в таблице?","Разобьём квадрат 5 * 5 без центральной клетки на четыре прямоугольника 2 * 3, а каждый из них разобьём на два прямоугольника 1 * 3. Получится 8 прямоугольников 1 * 3, сумма чисел в каждом из которых равна 23. Поскольку сумма вообще всех чисел равна 200, находим число в центральной клетке как 200 − 23 * 8 = 16. ",16,school,10 +"Известно, что (𝑎 + 𝑏) / (𝑎 - 𝑏) = 3. Найдите значение выражения (𝑎**2 - 𝑏**2) / (𝑎**2 + 𝑏**2)","Домножив равенство (𝑎 + 𝑏) / (𝑎 − 𝑏) = 3 на знаменатель, получим 𝑎 + 𝑏 = 3𝑎 − 3𝑏. Перенеся 𝑎 направо, а 3𝑏 налево, получим 4𝑏 = 2𝑎, откуда 𝑎 = 2𝑏. Подставив 𝑎 = 2𝑏 во второе выражение, получим (4𝑏**2 − 𝑏**2) / (4𝑏**2 + 𝑏**2) = 0,6.","0,6",school,10 +"У Юры есть 𝑛 карточек, на которых написаны числа от 1 до 𝑛. После того, как Юра потерял одну из них, сумма чисел на оставшихся оказалась равна 101. Какое число написано на потерянной карточке?","Предположим, что 𝑛 <= 13. Тогда 1 + 2 + … + 𝑛 = 𝑛(𝑛 + 1) 2 <= 91 < 101. Какую бы карточку Юра ни потерял, в любом случае общая сумма получится меньше 101, противоречие. Предположим 𝑛 >= 15. Потерянная карточка Юры содержит число не больше 𝑛, поэтому сумма оставшихся карточек не меньше 1 + 2 + … + (𝑛 − 1) = 𝑛(𝑛 − 1) / 2 >= 105 > 101, противоречие. Следовательно, 𝑛 = 14, а потерянное число равно 1 + 2 + … + 14 − 101 = 105 − 101 = 4",4,school,10 +"В центральной клетке доски 21 * 21 находится фишка. За один ход можно передвинуть фишку в соседнюю по стороне клетку. Алина сделала 10 ходов. Сколько существует клеток, где может оказаться фишка?","Покрасим всю доску в шахматную раскраску так, чтобы центральная клетка доски была чёрной. При передвижении фишки в соседнюю по стороне клетку каждый раз будет меняться цвет клетки, на которой стоит фишка. Спустя нечётное число ходов фишка всегда оказывается на белой клетке, а спустя чётное число ходов — на чёрной. Следовательно, после 10 ходов фишка точно окажется на чёрной клетке. Покажем, что во все чёрные клетки, в которые возможно попасть не более, чем за 10 ходов, можно попасть и ровно за 10 ходов. Рассмотрим произвольную чёрную клетку 𝐴, в которую можно попасть менее чем за 10 ходов из центральной. Поскольку к моменту попадания в клетку 𝐴 было сделано чётное количество ходов, меньшее 10, то дальше можно просто передвигать фишку в соседнюю клетку и обратно, пока не будет сделано ровно 10 ходов. Значит, нам надо посчитать количество чёрных клеток, в которые можно попасть не более чем за 10 ходов. За 0 ходов можно попасть только в начальную клетку, за 2 хода можно попасть в начальную клетку и в 4 * 2 = 8 новых клеток, за 4 хода можно попасть в те клетки, в которые уже попадали, и в 4 * 4 = 16 новых клеток и так далее. Получаем, что количество клеток, в которые можно попасть не более чем за 10 шагов, равно 1 + 8 + 16 + 24 + … + 40 = 1 + 8(1 + 2 + 3 + 4 + 5) = 121.",121,school,10 +"У Олега есть четыре карточки, на каждой из которых с одной и с другой стороны написаны натуральные числа (всего написано 8 чисел). Он рассматривает всевозможные четвёрки чисел, где первое число написано на первой карточке, второе — на второй, третье — на третьей, четвёртое — на четвёртой. Затем для каждой четвёрки он выписывает произведение чисел к себе в блокнот. Чему равна сумма восьми чисел на карточках, если сумма шестнадцати чисел в блокноте Олега равна 330?","Обозначим числа на одной карточке за 𝑎 и 𝑏, на другой — за 𝑐 и 𝑑, на третьей — за 𝑒 и 𝑓, на четвёртой— за 𝑔 и ℎ. По условию сумма 16 слагаемых вида 𝑎𝑐𝑒𝑔 + 𝑎𝑐𝑒ℎ + … + 𝑏𝑑𝑓ℎ равна 330. Заметим, что эта же сумма получается при раскрытии всех скобок в выражении (𝑎 + 𝑏)(𝑐 + 𝑑)(𝑒 + 𝑓)(𝑔 + ℎ). Следовательно, (𝑎 + 𝑏)(𝑐 + 𝑑)(𝑒 + 𝑓)(𝑔 + ℎ) = 330 = 2 * 3 * 5 * 11. Поскольку все числа являются натуральными, каждая из скобок больше 1. Значит, скобки равны числам 2, 3, 5, 11 в некотором порядке. Тогда их сумма равна 𝑎 + 𝑏 + 𝑐 + 𝑑 + 𝑒 + 𝑓 + 𝑔 + ℎ = 2 + 3 + 5 + 11 = 21",21,school,10 +"Двое рабочих за два часа вырыли траншею. При этом первый рабочий устал и начал работать втрое медленней, а второй рабочий раззадорился и начал работать втрое быстрее, так что на прокладку второй такой траншеи у них ушёл один час. Во сколько раз производительность второго превосходила производительность первого изначально? ","Пусть изначально производительность первого рабочего была равна x, а производительность второго рабочего была равна y. Заметим, что при прокладке второй траншеи суммарная производительность рабочих была вдвое выше изначальной, поэтому 2(x + y) = 1/3 x + 3y; y = 5/3 x. Значит, производительность второго была в 5/3 раза выше производительности первого",5/3,school,10 +Замок Персиваля имел квадратную форму. Однажды Персиваль решил расширить свои владения и добавил к замку квадратную пристройку. В результате периметр замка увеличился на 10%. На сколько процентов увеличилась площадь замка?,"Пусть ширина замка равна a, а ширина пристройки — b. Тогда первоначальный периметр равен 4a, а итоговый периметр равен 4a + 2b. Тогда: 1,1 * 4a = 4a + 2b следовательно b = 0,2a . Отсюда площадь замка стала равна a**2 + (0,2a)**2 = 1,04a**2 , то есть площадь увеличилась на 4%.",4,school,10 +"Известно, что a**2 + b = b**2 + c = c**2 + a. Какие значения может принимать выражение a(a**2 − b**2) + b(b**2 − c**2) + c(c**2 − a**2)?","Заметим, что равенство a**2 + b = b**2 + c можно записать в виде: a**2 − b**2 = c − b. Аналогично имеем b**2 − c**2 = a − c, c**2 − a**2 = b − a. Подставляя эти равенства в искомые выражения, получаем, что a(a**2 − b**2) + b(b**2 − c**2) + c(c**2 − a**2) = a(c − b) + b(a − c) + c(b − a) = 0",0,school,10 +"Несколько мудрецов построилось в колонну. На всех были либо черные, либо белые колпаки. Оказалось, что среди любых 10 подряд идущих мудрецов поровну мудрецов с белыми и с черными колпаками, а среди любых 12 подряд идущих — не поровну. Какое наибольшее количество мудрецов могло быть?","Докажем, что больше 15 мудрецов быть не может. Предположим противное, пусть мудрецов хотя бы 16. Последовательно занумеруем всех мудрецов. Рассмотрим девять подряд идущих мудрецов. Если к ним добавить одного из двух соседних мудрецов, то среди них будет одинаковое число мудрецов с белыми и чёрными колпаками, поэтому на любых мудрецах, между которыми находится 9 мудрецов, надеты колпаки одинакового цвета. Без ограничения общности, на первом мудреце надет чёрный колпак. Тогда на одиннадцатом мудреце также чёрный колпак. Если на двенадцатом мудреце надет белый колпак, то среди первых двенадцати мудрецов будет поровну белых и чёрных колпаков. Поэтому на двенадцатом мудреце надет чёрный колпак, откуда и на втором мудреце надет чёрный колпак. Аналогично рассмотрев мудрецов со второго по одиннадцатого, получим что на мудрецах 3 и 13 надеты колпаки чёрного цвета. Рассмотрев мудрецов с третьего по двенадцатого, получим, что на мудрецах 4 и 14 надеты колпаки чёрного цвета. Аналогично на мудрецах 5 и 15, 6 и 16 надеты колпаки чёрного цвета. Но тогда среди первых десяти мудрецов на первых шести чёрные колпаки, поэтому чёрных колпаков будет больше. Противоречие. 15 мудрецов может быть: пусть на первых 5 и последних 5 мудрецах надеты чёрные колпаки, а на оставшихся 5 надеты белые колпаки. Несложно понять, что тогда условие задачи будет выполнено.",15,school,10 +"Участвуя в шахматном турнире, Вася сыграл 52 партии. По старой системе подсчёта очков (1 очко за победу, 1/2 очка за ничью и 0 очков за поражение) он набрал 35 очков. Сколько очков он набрал по новой системе подсчёта очков (1 очко за победу, 0 очков за ничью и –1 очко за поражение)?","Пусть Вася в турнире a раз победил, b раз сыграл вничью и c раз проиграл. Тогда a + b + c = 52, a + b/2 = 35. Нужно найти значение a – c. Из второго соотношения следует, что b = 70 – 2a. Тогда a + (70 – 2a) + c = 52, откуда 70 + c – a = 52, a – c = 18.",18,school,10 +"Петя показал Васе 37 внешне одинаковых карточек, выложенных в ряд. Он сказал, что на закрытых сторонах карточек записаны все числа от 1 до 37 (каждое по одному разу) так, что число на любой карточке начиная со второй является делителем суммы чисел, написанных на всех предшествующих карточках. Затем Петя показал Васе, что на первой карточке написано число 37, а на второй — число 1. Вася сказал, что он тогда знает, какое число написано на третьей карточке. Какое?","Сумма всех чисел, кроме последнего, делится на последнее число, значит, сумма всех чисел также делится на последнее число. Сумма всех чисел от 1 до 37 равна 19 * 37. Значит, последнее число равно 1, 19 или 37. Так как 1 и 37 стоят на первом и втором местах, последнее число — 19. Третье число — делитель числа 37 + 1 = 38, то есть оно равно 1, 2 или 19. Мы знаем, что числа 1 и 19 расположены не на третьем месте, поэтому на третьем месте стоит число 2",2,school,10 +"Петя сбегает с четвёртого этажа на первый на 2 секунды быстрее, чем мама едет на лифте. Мама едет на лифте с четвёртого этажа на первый на 2 секунды быстрее, чем Петя сбегает с пятого этажа на первый. За сколько секунд Петя сбегает с четвёртого этажа на первый? (Длины пролетов лестницы между всеми этажами одинаковы).","Между первым и четвертым этажами 3 пролета, а между пятым и первым – 4. Согласно условию, Петя 4 пролета пробегает на 2 секунды дольше, чем мама едет на лифте, а три пролета – на 2 секунды быстрее мамы. Значит, за 4 секунды Петя пробегает один пролет. Тогда с четвертого этажа на первый (т.е. на 3 пролета) Петя сбегает за 4 * 3 = 12 секунд.",12,school,10 +"На числовой прямой закрашивают красным и синим цветом точки с целыми координатами по следующим правилам: а) точки, разность координат которых равна 7, должны быть покрашены одним цветом; б) точки с координатами 20 и 14 должны быть покрашены красным, а точки с координатами 71 и 143 — синим. Сколькими способами можно раскрасить все точки с целыми координатами, соблюдая эти правила?","Из пункта а) следует, что раскраска всех точек с целыми координатами однозначно определяется раскраской точек, соответствующих числам 0, 1, 2, 3, 4, 5 и 6. Точка 0 = 14 * 2 * 7 должна быть покрашена так же как 14, т.е. красным. Аналогично, точка 1 = 71 - 10 * 7 должна быть покрашена синим, точка 3 = 143 - 20 * 7 – синим, и 6 = 20 - 2 * 7 – красным. Поэтому остается только посчитать, сколькими различными способами можно раскрасить точки, соответствующие числам 2, 4 и 5. Так как каждую точку можно раскрасить двумя способами – красным или синим – то всего способов 2 * 2 * 2 = 8.",8,school,10 +"Каждый из 10 гномов либо всегда говорит правду, либо всегда лжет. Известно, что каждый из них любит ровно один сорт мороженого: сливочное, шоколадное или фруктовое. Сначала Белоснежка попросила поднять руки тех, кто любит сливочное мороженое, и все подняли руки, потом тех, кто любит шоколадное мороженое – и половина гномов подняли руки, потом тех, кто любит фруктовое мороженое – и руку поднял только один гном. Сколько среди гномов правдивых?","Гномы, которые всегда говорят правду, подняли руку один раз, а гномы, которые всегда лгут, – два раза. Всего было поднято 16 рук (10 + 5 + 1). Если бы все гномы сказали правду, то было бы поднято 10 рук. Если одного правдивого гнома заменить на одного лгуна, то число поднятых рук увеличится на 1. Так как было поднято 6 «лишних» рук, то 6 гномов солгали, а 4 сказали правду",4,school,10 +"По кругу стоят 100 белых точек. Аня и Боря красят по очереди по одной еще не покрашенной точке в красный или синий цвет, начинает Аня. Аня хочет, чтобы в итоге оказалось как можно больше пар разноцветных соседних точек, а Боря — чтобы оказалось как можно меньше таких пар. Какое наибольшее число пар разноцветных соседних точек Аня может гарантировать себе независимо от игры Бори?","Нужно показать, что Аня всегда может добиться, чтобы разноцветных пар было не меньше 50, а Боря сможет помешать ей добиться, чтобы таких пар было больше 50. Стратегия Ани. Первым ходом Аня красит в любой цвет любую точку, а дальше каждым ходом выбирает пару из непокрашенной точки и стоящей рядом с ней покрашенной (такая, очевидно, найдется), и красит непокрашенную точку в цвет, отличный от цвета покрашенной. При этом образуется новая пара соседних разноцветных точек. Стратегия Бори. Каждым ходом Боря выбирает пару из непокрашенной точки и стоящей рядом с ней покрашенной, и красит непокрашенную точку в цвет, совпадающий с цветом покрашенной. При этом образуется новая пара соседних одноцветных точек. Обоснование правильности стратегий. Всего в круге имеется 100 пар соседних точек, и каждый игрок делает за игру по 50 ходов. Сделав свои ходы, Боря добьется того, что из этих 100 пар хотя бы 50 будут одноцветными, а Аня — что хотя бы 49 из них будут разноцветными. Однако заметим, что количество разноцветных пар всегда четно. Действительно, после окончания игры пройдем полный круг, начиная с какой-то отмеченной точки (пусть для определенности с красной). Группы из идущих подряд красных и синих точек при этом будут чередоваться: К—С—К—С—. . . —К, и значит, встретим пар разноцветных соседей вида K—C столько же, сколько пар вида С—К. Поэтому если пар разноцветных соседних точек не меньше 49, то их хотя бы 50. +",50,region,10 +"У Васи есть n конфет нескольких сортов, где n > 145. Известно, что если из данных n конфет выбрать любую группу, содержащую не менее 145 конфет (в частности, можно выбрать группу из всех данных n конфет), то существует такой сорт конфет, что выбранная группа содержит в точности 10 конфет этого сорта. Найдите наибольшее возможное значение n.","Оценка. Докажем, что n > 160 «не работает». Пусть дан набор из n конфет. Назовем сорт критическим, если конфет этого сорта ровно 10 (среди всех данных n конфет). Пусть у нас k критических сортов, тогда всего конфет не менее 10k: n >= 10k. Уберем по одной конфете каждого критического сорта и организуем группу из оставшихся n − k конфет. Для этой группы нет сорта, представленного ровно 10 конфетами. Кроме того, n − k > n − n/10 = 9n/10 >= (9 * 160) / 10 = 144. Значит, в рассматриваемой группе не менее 145 конфет, поэтому условие задачи не выполняется. Пример. Теперь привед¨ем пример ситуации, в которой у Васи может быть 160 конфет. Пусть у него есть ровно по 10 конфет 16 сортов. Пусть выбрана группа, для которой нет сорта, представленного ровно 10 конфетами. Тогда в эту группу не входит хотя бы одна конфета каждого сорта (иначе говоря, ни один сорт не будет взят полностью), т.е. группа содержит не более 16 * 9 = 144 конфет, значит, условие задачи выполнено",160,region,10 +"На доску выписали три натуральных числа: два десятизначных числа a и b, а также их сумму a+b. Какое наибольшее количество нечетных цифр могло быть выписано на доске?","Заметим, что в числе a + b не более 11 разрядов, таким образом всего на доске выписано не более 31 цифры. При этом все три числа a, b, a + b не могут оказаться одновременно неч¨етными. Следовательно, одна из их тр¨ех последних цифр — ч¨етная, поэтому неч¨етных цифр выпсано не более 30. Привед¨ем пример, показывающий, что неч¨етных цифр могло оказаться ровно 30: 5 555 555 555 + 5 555 555 555 = 11 111 111 110.",30,region,10 +"Олег нарисовал пустую таблицу 50 * 50 и написал сверху от каждого столбца и слева от каждой строки по ненулевому числу. Оказалось, что все 100 написанных чисел различны, причём 50 из них рациональные, а остальные 50 — иррациональные. Затем в каждую клетку таблицы он записал произведение чисел, написанных около её строки и её столбца («таблица умножения»). Какое наибольшее количество произведений в этой таб- лице могли оказаться рациональными числами?","Сначала покажем, что иррациональных чисел в таблице не меньше 1250. Пусть вдоль левой стороны таблицы выписано x иррациональных и 50 − x рациональных чисел. Тогда вдоль верхней стороны выписаны 50 − x иррациональных и x рациональных чисел. Поскольку произведение ненулевого ра- ционального и иррационального чисел всегда иррационально, в таблице стоит хотя бы x**2+(50−x)**2 иррациональных чисел. При этом x**2 + (50 − x)**2 = 2x**2 − 100x + 502 = 2(x − 25)**2 + 2 * 25**2 > > 2 * 25**2 = 1250, что и требовалось. Отсюда следует, что в таблице не более 2500 − 1250 = 1250 рациональных чисел. Ровно 1250 рациональных чисел в таблице может быть, например, в таком случае. Вдоль левой стороны стоят числа 1, 2, . . . , 24, 25, 2**(1/2), 2*(2**(1/2)), . . . , 25*(2)**(1/2), а вдоль верхней стороны — числа 26, 27, . . . , 49, 50, 26*(2**(1/2)), 27*(2**(1/2)), . . . , 50*(2**(1/2)). Тогда иррациональными будут только 2 * 25**2 = 1250 произведений рационального и иррационального чисел",1250,region,10 +"Даны квадратные трёхчлены f1(x), f2(x), . . . , f100(x) с одинаковыми коэффициентами при x 2 , одинаковыми коэффициентами при x, но различными свободными членами; у каждого из них есть по два корня. У каждого трёхчлена fi (x) выбрали один корень и обозначили его через xi . Какие значения может принимать сумма f2(x1) + f3(x2) + . . . + f100(x99) + f1(x100)?","Пусть i-й трёхчлен имеет вид fi (x) = ax**2+bx+ci . Тогда f2(x1) = a(x1)**2 + bx1 + c2 = (a(x1)**2 + bx1 + c1) + (c2 − c1) = c2 − c1, поскольку f1(x1) = 0. Аналогично получаем равенства f3(x2) = = c3 − c2, . . . , f100(x99) = c100 − c99 и f1(x100) = c1 − c100. Складывая полученные равенства, получаем f2(x1) + f3(x2) + . . . + f1(x100) = (c2 − c1) + . . . + (c1 − c100) = 0. Значит, единственное возможное значение суммы — ноль.",0,region,10 +"После просмотра фильма зрители по очереди оценивали фильм целым числом баллов от 0 до 10. В каждый момент времени рейтинг фильма вычислялся как сумма всех выставленных оценок, делённая на их количество. В некоторый момент времени T рейтинг оказался целым числом, а затем с каждым новым проголосовавшим зрителем он уменьшался на единицу. Какое наибольшее количество зрителей могло проголосовать после момента T?","Рассмотрим некоторый момент, когда рейтинг уменьшился на 1. Пусть перед этим проголосовало n человек, и рейтинг был целым числом x. Значит, сумма баллов стала равна nx. Пусть следующий зритель выставил y баллов. Тогда сумма баллов стала равна nx+y = (n+1)(x−1), откуда y = x−n−1. Наибольшее возможное значение x равно 10, а наименьшее возможное значение n равно 1; значит, наибольшее значение y (на первом таком шаге) равно 8. С каждым следующим шагом значение x уменьшается на 1, а значение n увеличивается на 1. Следовательно, на втором шаге значение y не превосходит 6, на третьем — 4, и т.д. Поскольку любая оценка не меньше 0, число шагов не превосходит 5. Осталось показать, что пять шагов возможны. Пусть рейтинг в момент T равен 10 (при одном проголосовавшем), затем второй зритель выставляет 8 баллов, третий — 6, четвёртый — 4, пятый — 2, а шестой — 0. Тогда рейтинг последовательно принимает значения 9, 8, 7, 6 и 5.",5,region,10 +"Девять действительных a1, a2, ..., a9 образуют арифметическую прогрессию. Известно, что a9 в 3 раза больше среднего арифметического этих девяти чисел. Найдите a1, если известно, что a4 = 6.","Пусть 𝑎 — первый член прогрессии, а 𝑑 — её разность, тогда девять члено�� прогрессии равны 𝑎, 𝑎 + 𝑑, 𝑎 + 2𝑑, …, 𝑎 + 8𝑑. Среднее арифметическое чисел в арифметической прогрессии, состоящей из нечётного числа членов, равно среднему из этих чисел, т. е. в данном случае 𝑎 + 4𝑑. Получаем уравнение 𝑎 + 8𝑑 = 3(𝑎 + 4𝑑), откуда следует 𝑎 + 8𝑑 = 3𝑎 + 12𝑑 и 𝑎 = −2𝑑. Тогда 6 = 𝑎4 = 𝑎 + 3𝑑 = 𝑑. Значит, 𝑎1 = −2𝑑 = −12.",-12,school,11 +"В сосуде, имеющем форму правильной треугольной призмы, находилась вода, причём её уровень составлял 30 сантиметров. Всю эту воду перелили в пустой сосуд, имеющий форму правильной шестиугольной призмы, сторона основания которой вдвое меньше стороны основания треугольной призмы. Чему равен уровень воды теперь? Ответ выразите в сантиметрах","В обоих случаях часть, занятая водой, имеет форму правильной призмы: в одном случае треугольной, в другой — шестиугольной. Объём правильной призмы, одинаковый в обоих случаях, равен произведению площади основания на высоту. Следовательно, отношение высот призм (отношение уровней воды) обратно отношению площадей их оснований. Если бы шестиугольная призма имела такую же сторону основания, как и треугольная, её площадь основания была бы в 6 раз больше, т. к. правильный шестиугольник можно разбить на 6 правильных треугольников с такой же стороной. При уменьшении стороны в 2 раза площадь уменьшается в 2**2 = 4 раза, поэтому площадь основания шестиугольной призмы в 6/4 = 1,5 раза больше площади основания треугольной призмы. Но тогда уровень воды в ней в 1,5 раза меньше, т. е. он составляет 30/1,5 = 20 сантиметров.",20,school,11 +"Андрей, Борис и Влад зашли в магазин. Андрей купил 1 мороженое, 2 булочки и 3 шоколадки и заплатил за это 235 рублей. Борис купил 3 порции мороженого, 2 булочки и 1 шоколадку и заплатил за это 205 рублей. Сколько рублей должен будет заплатить Влад, если он купит 6 порций мороженого, 5 булочек и 4 шоколадки?","Если сложить покупки Андрея и Бориса, то мы получим, что 4 порции мороженого, 4 булочки и 4 шоколадки стоят 235 + 205 = 440 рублей. Отсюда можно понять, что 1 мороженое, 1 булочка и 1 шоколадка стоят суммарно 440/4 = 110 рублей. Если теперь это умножить на 3 и добавить покупку Бориса, то мы как раз получим 6 порций мороженого, 5 булочек и 4 шоколадки. Значит, искомая стоимость равна 110 * 3 + 205 = 535 рублей.",535,school,11 +"Найдите наибольшее натуральное число, которое в 9 раз больше своего остатка от деления на 1024.","Обозначим неполное частное от деления исходного числа на 1024 через 𝑘, а остаток от деления — через 𝑟. Тогда исходное число равно 1024𝑘 + 𝑟, а также по условию это равно 9𝑟. Отсюда получаем 8𝑟 = 1024𝑘, т. е. 𝑟 = 128𝑘. Поскольку 𝑟 — остаток от деления на 1024, то 𝑟 < 1024. Наибольшее число, делящееся на 128 и меньшее 1024 = 128 * 8, равно 128 * 7. Значит, наибольшее возможное исходное число равно 9𝑟 = 9 * 128 * 7 = 8064 — ясно, что оно подходит под условие задачи",8064,school,11 +"В стране 15 городов. Между каждыми двумя из них либо есть дорога, либо её нет. Оказалось, ��то для любого города A найдутся такие три города, что они между собой попарно не соединены дорогами, но каждый из них соединён дорогой с A. Какое наибольшее количество дорог может быть в этой стране?","Рассмотрим произвольный город A. По условию найдутся города B, C, D такие, что они соединены с A, но попарно не соединены между собой. В частности, это означает, что C и D не соединены с B. Теперь рассмотрим город B. Для него тоже должны найтись соответствующие 3 города. Заметим, что это не могут быть города C и D, поскольку они не соединены с B. Значит, должны найтись города K , L и M, попарно не соединённые между собой (среди них, возможно, есть город 𝐴). Но тогда в стране нет хотя бы 6 дорог: BC, CD, BD, 𝐾𝐿, 𝐿𝑀, 𝐾𝑀. Всего возможных дорог в стране 1/2 * 15 * 14 = 105 , из которых хотя бы 6 отсутствует. Значит, всего дорог не более 105 - 6 = 99 . Теперь поймём, что дорог может быть ровно 99. Пронумеруем города числами от 1 до 15. Пусть города 1, 2, 3 попарно не соединены между собой, а также города 4, 5, 6 попарно не соединены между собой, а любые другие пары городов дорогой соединены. Проверим, что условие задачи выполняется. Действительно, если в качестве города 𝐴 из условия задачи взять город из первой тройки, то для него подходит тройка городов 4, 5, 6. Если же в качестве города 𝐴 из условия взять город не из первой тройки, то для него подходит тройка городов 1, 2, 3.",99,school,11 +"Маша живёт в квартире №290, которая находится в 4-м подъезде 17-этажного дома. На каком этаже живёт Маша? (Количество квартир одинаково во всех подъездах дома на всех 17 этажах; номера квартир начинаются с 1.)","Обозначим за 𝑥 количество квартир на этаже, тогда в каждом подъезде 17𝑥 квартир. Таким образом, в первых трёх подъездах будет 51𝑥 квартир, а в первых четырёх — 68𝑥. Если 𝑥 >= 6, то в первых трёх подъездах хотя бы 306 квартир, поэтому квартира №290 не может располагаться в четвёртом подъезде. А если 𝑥 <= 4, то в первых четырёх подъездах будет не больше 272 квартир, то есть снова квартира №290 не может располагаться в четвёртом подъезде. Остаётся единственный вариант, когда 𝑥 = 5. Тогда в первых трёх подъездах 255 квартир, а квартира №290 является 35-й в четвёртом подъезде, т. е. расположена на 7-м этаже.",7,school,11 +"На столе лежат 30 монет: 23 десятирублёвых и 7 пятирублёвых, причём 20 из этих монет лежат вверх орлом, а остальные 10 — решкой. При каком наименьшем 𝑘 среди произвольно выбранных 𝑘 монет обязательно найдётся десятирублёвая монета, лежащая орлом вверх?","Если выбрать 18 монет, то среди них окажется не более 10 лежащих решкой вверх, поэтому хотя бы 8 монет будут лежать орлом вверх. Среди этих монет не более 7 пятирублёвых, поэтому хотя бы одна будет десятирублёвой, она-то нам и подойдёт. С другой стороны, если исходно на столе лежат 7 пятирублёвых монет орлом вверх, 10 десятирублёвых монет решкой вверх и 13 десятирублёвых монет орлом вверх, то среди 17 монет могут оказаться только монеты первых двух типов, поэтому 17 монет (или меньше) может не хватить.",18,school,11 +"Произведение положительных чисел 𝑎 и 𝑏 равно 1. Известно, что (3𝑎 + 2𝑏)(3𝑏 + 2𝑎) = 295. Найдите 𝑎 + 𝑏.","Раскрыв скобки, получаем 295 = 6𝑎**2 + 6𝑏**2 + 13𝑎𝑏 = 6(𝑎**2 + 𝑏**2) + 13, откуда 𝑎**2 + 𝑏**2 = 47. Тогда +(𝑎 + 𝑏)**2 = 𝑎**2 + 𝑏**2 + 2𝑎𝑏 = 47 + 2 = 49 = 7**2, что даёт 𝑎 + 𝑏 = 7 (отметим, что 𝑎 + 𝑏 > 0, поскольку 𝑎 > 0 и 𝑏 > 0). ",7,school,11 +"При каком наименьшем натуральном 𝑛 можно расставить числа от 1 до 𝑛 по кругу так, чтобы каждое число было либо больше всех 40 следующих за ним по часовой стрелке, либо меньше всех 30 следующих за ним по часовой стрелке?","Если 𝑛 <= 39, то для числа 𝑛 условие выполняться не может: оно не может быть ни больше 40 следующих за ним чисел (ведь оно не больше самого себя), ни меньше 30 следующих за ним чисел (ведь оно наибольшее). Если 40 <= 𝑛 <= 69, то для числа 40 условие выполняться не может: нет ни 40 чисел, меньших его, ни 30 чисел, больших его. Если же 𝑛 = 70, то числа расставить получится. Для этого можно поставить их в следующем порядке по часовой стрелке: 1, 2, 3, …, 40, 70, 69, 68, …, 41. Тогда числа от 1 до 40 окажутся меньше следующих 30 за ними, а числа от 70 до 41 — больше следующих 40 за ними.",70,school,11 +"У многочлена 𝑃(𝑥) все коэффициенты — целые неотрицательные числа. Известно, что 𝑃(1) = 4 и 𝑃(5) = 152. Чему равно 𝑃(11)?","Можно заметить, что 𝑃(1) = 4 — это в любом случае сумма коэффициентов многочлена 𝑃(𝑥) = 𝑎𝑛𝑥**𝑛 +… + 𝑎1𝑥 + 𝑎0 , откуда каждый из них не превосходит 4. Тогда эти коэффициенты можно считать цифрами в пятеричной системе счисления, а значение многочлена в точке 5 — как раз число, записанное этими цифрами: 152 = 𝑃(5) = (5**𝑛)𝑎𝑛 + … + 5𝑎1 + 𝑎0 . Так как 15210 = 11025 , а представления чисел в пятеричной системе единственны, то 𝑃(𝑥) = 1𝑥**3 + 1𝑥**2 + 0𝑥 + 2.",1454,school,11 +Произведение девяти последовательных натуральных чисел делится на 1111. Какое наименьшее возможное значение может принимать среднее арифметическое этих девяти чисел?,"Пусть эти девять чисел — 𝑛, 𝑛+1, … , 𝑛+8 для некоторого натурального 𝑛. Ясно, что их среднее арифметическое равно 𝑛 + 4. Чтобы произведение делилось на 1111 = 11 * 101, необходимо и достаточно, чтобы хотя бы один из множителей делился на 11, а также хотя бы один из множителей делился на 101. Тогда какое-то из девяти чисел не меньше 101, поэтому 𝑛 + 8 >= 101 и 𝑛 + 4 >= 97. Ясно также, что для девяти чисел 93, 94, … , 101 значение 𝑛+4 = 97 достигается, ведь 99 делится на 11, а 101 делится на 101.",97,school,11 +"В турнире по футболу участвовало 15 команд, каждая сыграла с каждой ровно один раз. За победу давалось 3 очка, за ничью — 1 очко, а за поражение — 0 очков. После завершения турнира оказалось, что некоторые 6 команд набрали хотя бы 𝑁 очков каждая. Какое наибольшее целое значение может принимать 𝑁?","Назовём эти 6 команд успешными, а остальные 9 команд назовём неуспешными. Назовём игру двух успешных команд внутренней, а игру успешной и неуспешной команды — внешней. Сразу заметим, что за каждую игру участвующие в ней команды суммарно получают не более 3 очков. Ясно, что внутренних игр было ровно (6 * 5) / 2 = 15, и только за такие игры все успешные команды суммарно заработали не более 15 * 3 = 45 очков. Внешних же игр было ровно 6 * 9 = 54, и в каждой такой игре успешная команда зарабатывала не более 3 очков. Итого за внешние игры все успешные команды суммарно набрали не более 54 * 3 = 162 очка. По условию успешные команды суммарно набрали хотя бы 6𝑁 очков, поэтому получаем неравенство 6𝑁 <= 45 + 162. Из него следует, что 𝑁 <= 207/6 < 35 и 𝑁 <= 34. Теперь приведём пример для 𝑁 = 34. Пронумеруем команды числами от 1 до 15. Покажем, как команды от 1 до 6 могут набрать хотя бы 34 очка. Пусть каждая команда от 1 до 6 выиграла у каждой команды от 7 до 15, тогда только за такие игры каждая команда от 1 до 6 набрала 9 * 3 = 27 очков. Пусть в каждой игре команд от 7 до 15 выиграла команда с большим номером (исход этих игр не имеет значения). +Итого команды от 1 до 6 набрали хотя бы 27 + 7 = 34 очка.",34,school,11 +"В стране 110 городов. Между каждыми двумя из них либо есть дорога, либо её нет. Автомобилист находился в некотором городе, из которого вела ровно одна дорога. Проехав по дороге, он оказался во втором городе, из которого вели уже ровно две дороги. Проехав по одной из них, он оказался в третьем городе, из которого вели уже ровно три дороги, и так далее. В какой-то момент, проехав по одной из дорог, он оказался в 𝑁-м городе, из которого вели уже ровно 𝑁 дорог. На этом автомобилист своё путешествие прекратил. (Для каждого 2 <= 𝑘 <= 𝑁 из 𝑘-го города выходило ровно 𝑘 дорог с учётом той, по которой автомобилист в этот город приехал.) Какое наибольшее значение может принимать 𝑁?","Пронумеруем города в порядке их посещения автомобилистом: 1, 2, 3, … , 𝑁. Предположим, 𝑁 >= 108. Из города 1 ведёт дорога только в город 2, поэтому из города 108 все дороги ведут во все 108 городов, кроме 1 и 108. Но тогда из города 2 ведёт хотя бы три дороги: в города 1, 3 и 108. Противоречие, значит, 𝑁 <= 107. Приведём пример для 𝑁 = 107. Пусть города имеют номера 1, 2, 3, … , 109, 110. Пусть для всех 1 <= 𝑖 <= 54 есть дорога из города 𝑖 в город 𝑖 + 1; для всех 56 <= 𝑗 <= 107 есть дороги из города 𝑗 во все города с 110 − 𝑗 по 110 включительно, кроме самого города 𝑗; из города 55 есть дороги в города 109 и 110; никаких других дорог в стране нет. Несложно убедиться, что для всех 1 <= 𝑘 <= 107 из города 𝑘 ведёт ровно 𝑘 дорог. По этим городам и мог проехать автомобилист.",107,school,11 +"Внутри круга нарисовано 16 радиусов этого круга и 10 окружностей, центры которых совпадают с центром круга. На сколько областей радиусы и окружности делят круг?","10 окружностей разбивают круг на 10 колец и один меньший круг, всего 11 частей. 16 радиусов разбивают каждую из 11 частей ещё на 16. Всего получается 11 * 16 = 176 областей.",176,school,11 +"Вдоль дороги в один ряд стоят 25 столбов. Иногда на один из столб��в садится чиж, и сразу же с одного из соседних столбов взлетает чиж (если на соседних столбах в этот момент хоть кто-нибудь сидел). Также на каждом столбе не может сидеть более одного чижа. Первоначально на столбах нет птиц. Какое наибольшее количество чижей могут одновременно находиться на столбах?","Сперва покажем, что все столбы занять не удастся. Предположим, такое произошло. Рассмотрим чижа, который сел последним. Поскольку он занял последний незанятый столб, рядом с ним обязательно был занятый столб. Следовательно, чиж, который сидел на этом столбе, должен был улететь. Противоречие. Теперь приведём пример того, как чижи могли занять 24 столба. Для удобства пронумеруем все столбы по порядку. Пусть первый чиж сядет на первый столб. Предположим, заняты все столбы с 1-го по 𝑘-й, где 𝑘 <= 23. Покажем, как получить ситуацию, где заняты все столбы с 1-го по (𝑘 + 1)-й. Последовательно меняя 𝑘 от 1 до 23, получим ситуацию, в которой будут заняты первые 24 столба. Пусть заняты все столбы с 1-го по 𝑘-й, пусть тогда следующий чиж садится на (𝑘 + 2)-й столб (где 𝑘 + 2 <= 25), а следующий за ним чиж садится на (𝑘 + 1)-й столб, и чиж с (𝑘 + 2)-го столба улетает. Получаем ситуацию, где заняты все столбы с 1-го по (𝑘 + 1)-й",24,school,11 +"Натуральное число 𝑛 назовём интересным, если 2𝑛 является точным квадратом, а 15𝑛 — точным кубом. Найдите наименьшее интересное число.","Разложим число 𝑛 на простые множители. Чтобы число было квадратом, необходимо, чтобы в этом разложении все простые числа встречались в чётных степенях, а чтобы число было кубом, необходимо, чтобы все простые числа встречались в делящихся на 3 степенях. Посмотрим, на какую степень двойки делится 𝑛. Во-первых, эта степень нечётная, т. к 2𝑛 — точный квадрат. Во-вторых, эта степень делится на 3, т. к. 15𝑛 — точный куб. Следовательно, минимальная степень двойки — это 3. Теперь посмотрим, на какую степень тройки делится 𝑛. Во-первых, эта степень чётная, т. к. 2𝑛 — точный квадрат. Во-вторых, эта степень даёт остаток 2 при делении на 3, т.к. 15𝑛 — точный куб. Следовательно, минимальная степень тройки — это 2. Для пятёрки аналогично получаем, что её минимальная степень — это 2. Следовательно, 𝑛 делится на 2**3 * 3**2 * 5**2 = 1800, т. е. 𝑛 >= 1800. Несложно проверить, что 𝑛 = 1800 удовлетворяет всем условиям задачи.",1800,school,11 +"По зову воеводы пришли 55 солдат: лучники и мечники. Все они были одеты либо в золотые, либо в чёрные доспехи. Известно, что мечники говорят правду, когда носят чёрные доспехи и обманывают, когда носят золотые доспехи, а лучники — наоборот. На вопрос «На тебе золотые доспехи?» утвердительно ответили 44 человека. +На вопрос «Ты лучник?» утвердительно ответили 33 человека. На вопрос «Сегодня понедельник?» утвердительно ответили 22 человека. Сколько пришло лучников в золотых доспехах на зов воеводы?","На первый вопрос утвердительно ответят лучники в золотых дос��ехах и лучники в чёрных доспехах, то есть все лучники. На второй вопрос утвердительно ответят лучники в золотых доспехах и мечники в золотых доспехах, то есть все солдаты в золотых доспехах. На третий вопрос утвердительно ответят либо мечники в золотых доспехах и лучники в чёрных доспехах (если сегодня не понедельник), либо мечники в чёрных доспехах и лучники в золотых доспехах (если сегодня понедельник). В первом случае (если не понедельник) просуммируем количество утвердительных ответов на все три вопроса. Получится удвоенное количество мечников в золотых доспехах плюс удвоенное количество лучников, т. е. чётное число, но по условию оно равно 22 + 33 + 44 = 99 и является нечётным — противоречие. Значит, на третий вопрос утвердительно ответили мечники в чёрных доспехах и лучники в золотых доспехах. Просуммируем теперь количество утвердительных ответов. Получится утроенное количество лучников в золотых доспехах плюс количество всех остальных (по одному разу). Тогда, если вычесть из данного числа общее количество солдат, получится удвоенное количество лучников в золотых доспехах, которое надо поделить пополам: (22 + 33 + 44 − 55)/2 = 22",22,school,11 +"Внутри шляпы волшебника живут 100 кроликов: белые, синие и зелёные. Известно, что если произвольным образом вытащить из шляпы 81 кролика, то среди них обязательно найдутся три разноцветных. Какое наименьшее количество кроликов нужно достать из шляпы, чтобы среди них точно было два разноцветных?","Докажем, что если произвольным образом вытащить из шляпы 61 кролика, то среди них найдутся два разноцветных. Предположим противное: пусть имется a > 61 кроликов какого-то цвета (например, белого). Пусть второй цвет по количеству кроликов — синий. Тогда в шляпе живёт хотя бы (100−a)/2 синих кроликов. А значит, общее количество белых и синих хотя бы a + (100 − a)/2 = (100 + a)/2 >= 161/2 = 80,5 . Так как кроликов целое число, белых и синих вместе хотя бы 81, что противоречит условию. Покажем, что 60 кроликов может быть недостаточно. Пусть в шляпе живёт 60 белых и по 20 синих и зеленых. Тогда может получиться, что все вытащенные кролики белые. С другой стороны, если вытащить 81 кролика, то среди них точно встретятся кролики всех трёх цветов.",61,school,11 +"В трёхзначном числе первую цифру (разряд сотен) увеличили на 3, вторую — на 2, третью — на 1. В итоге число увеличилось в 4 раза. Приведите пример такого исходного числа.","Ответ может быть найден следующим способом. Пусть x — искомое число. Тогда условие задачи мгновенно приводит к уравнению x + 321 = 4x, единственным решением которого служит x = 107",107,school,11 +"Билет в кино стоил 300 рублей. Когда цену понизили, количество посетителей увеличилось на 50 процентов, а выручка кинотеатра выросла на 35 процентов. Сколько рублей составляет цена одного билета теперь?","Пусть цена нового билета составляет s рублей. Пусть изначально количество посетителей равнялось N, а после увеличения на 50% стало равняться 1,5N. Тогда по условию нынешняя выручка кинотеатра 1,5N * s на 35% больше, чем N * 300, откуда имеем 1,5Ns = 1,35 * N * 300, и s = 270.",270,school,11 +"Дана арифметическая прогрессия. Сумма первых её 10 членов равна 60, а сумма первых 20 её членов равна 320. Чему может быть равен 15-й член этой прогрессии?","Пусть первый член последовательности равен a, а разность равна b. Тогда сумма первых 10 её членов равна a + (a + b) + . . . + (a + 9b) = 10a+45b. Сумма первых двадцати членов равна a+(a+b)+. . .+(a+19b) = 20a + 190b. По условию 10a + 45b = 60, 20a + 190b = 320. Решая систему, находим a = −3, b = 2. Тогда 15-й член — это a + 14b = 25.",25,school,11 +"За лето однокомнатная квартира подорожала на 21%, двухкомнатная — на 11%, а суммарная стоимость квартир — на 15%. Во сколько раз однокомнатная квартира дешевле двухкомнатной?","Пусть однокомнатная квартира стоила a рублей, двухкомнатная — b рублей. Тогда из условия задачи следует, что 1,21a + 1,11b = 1,15(a + b), откуда 1,5a = b.",1.5,school,11 +"В турнире по шашкам участвовали ученики 10 и 11 классов. Каждый сыграл с каждым один раз. За победу участник получал 2 очка, за ничью — 1 очко, за проигрыш — 0 очков. Одиннадцатиклассников было в 10 раз больше, чем десятиклассников, и они вместе набрали в 4,5 раза больше очков, нежели все десятиклассники. Сколько очков набрал самый успешный десятиклассник?","1. Пусть в турнире приняло участие a десятиклассников, которые заработали b очков. Тогда играли 10a одиннадцатиклассников, которые заработали 4,5b очков. В каждой партии разыгрывают 2 очка, всего 11a игроков играют 11a(11a –1)/2 партий. Значит, из условия задачи следует соотношение 11a(11a – 1) = 5,5b, откуда b = 2a(11a – 1). 2. Заметим, что каждый участник играет 11a – 1 партий. Значит, каждый десятиклассник может набрать максимум 2(11a – 1) очков, если выиграет все игры. Так как a десятиклассников набрали 2a(11a – 1), они выиграли все свои игры. 3. Если в турнире участвовало хотя бы два десятиклассника, то в игре между собой один из них не выиграл. Это невозможно. Значит, был только один десятиклассник, т. е. a = 1. Он набрал 2(11a – 1) = 20 очков.",20,school,11 +"Пусть x1 < x2 < ... < x2024 — возрастающая последовательность натуральных чисел. При i = 1, 2, ..., 2024 обозначим pi = (x1 − 1/x1)(x2 − 1/x2)...(xi − 1/xi). Какое наибольшее количество натуральных чисел может содержаться среди чисел p1, p2, ..., p2024?","Приведем другой пример последовательности, дающий 2023 натуральных числа. Положим x1 = 2, x2 = = 3, ... , x2024 = 2025. Тогда xk − 1/xk = k + 1 − 1/(k + 1) = k(k + 2)/k + 1 . Тогда pi = (1 * 3)/2 * (2 * 4)/3 * ... * i(i + 2)/(i + 1) = i!(i + 2)/2, что является натуральным числом при i > 2.",2023,region,11 +На доске написано n различных целых чисел. Произведение двух наибольших равно 77. Произведение двух наименьших тоже равно 77. При каком наибольшем n это возможно?,"Числа −11, −7, −6, −5, . . . , 6, 7, 11 дают пример при n = 17. Допустим, что есть хотя бы 18 чисел с таким свойством. Тогда какие-то 9 из них будут одного знака (все положительны или все отрицательны). Среди этих 9 чисел модули двух наибольших будут не меньше 8 и 9 соответственно. Тогда их произведение не может быть равно 77",17,region,11 +"В некоторых клетках квадрата 200 * 200 стоит по одной фишке — красной или синей; остальные клетки пусты. Одна фишка видит другую, если они находятся в одной строке или одном столбце. Известно, что каждая фишка видит ровно пять фишек другого цвета (и, возможно, некоторое количество фишек своего цвета). Найдите наибольшее возможное количество фишек, стоящих в клетках.","Пример, содержащий 3800 фишек, получается, например, так. Выделим у квадрата 200 * 200 «каемку» ширины 5. Эта каемка состоит из четырех угловых квадратов 5 * 5 и четырех прямоугольников 5 * 190. Расставим фишки в эти четыре прямоугольника: в левый и в верхний — красные, а в правый и в нижний — синие. Нетрудно видеть, что этот пример удовлетворяет всем требованиям, и в нем по 1900 красных и синих фишек. Осталось доказать, что фишек не может быть больше 3800. Рассмотрим произвольную расстановку фишек, удовлетворяющую требованиям. Назовем ряд (строку или столбец) разноцветным, если в нем есть фишки обеих цветов. Сделаем сразу два полезных замечания. Во-первых, каждая фишка видит какую-то фишку другого цвета, поэтому каждая фишка лежит хотя бы в одном разноцветном ряду. Кроме того, поскольку разноцветный ряд содержит красную фишку, в нем не может быть более пяти синих фишек (иначе красная все их увидит). Аналогично, в разноцветном ряду не более пяти красных фишек, то есть всего не более 10 фишек. Теперь нетрудно получить требуемую оценку. Если есть 191 разноцветная строка, то в них не более 191 * 10 = 1910 фишек, а в оставшихся девяти строках не более 9 * 200 = 1800 фишек, итого не больше 1910 + 1800 < 3800 фишек. Аналогично разбирается случай, когда есть 191 разноцветный столбец. Если же и тех и других не более, чем по 190, то они содержат не более 190 * 10 + 190 * 10 = 3800 фишек, причем все фишки содержатся в этих рядах. Оценка доказана.",3800,allrus,11 +"На клетчатый лист бумаги размера 100 * 100 положили несколько попарно неперекрывающихся картонных равнобедренных прямоугольных треугольничков с катетом 1; каждый треугольничек занимает ровно половину одной из клеток. Оказалось, что каждый единичный отрезок сетки (включая граничные) накрыт ровно одним катетом треугольничка. Найдите наибольшее возможное число клеток, не содержащих ни одного треугольничка.","Положим n = 50. Назовем треугольничек верхним, если он расположен сверху от прямой, содержащей его горизонтальный катет, и нижним иначе. Пронумеруем горизонтальные линии сетки снизу вверх числами от 0 до 2n. Обозначим через uk (соответственно dk ) число отрезочков k-й линии, участвующих в верхних (соответственно нижних) треугольничках; тогда uk + dk = 2n и u0 = d2n = 2n. Кроме того, вертикальные отрезки сетки, расположенные между k-й и (k + 1)-й линиями, участвуют ровно в uk +dk+1 треугольничках, так что uk + dk+1 = 2n + 1. Отсюда несложно получить, что dk = k и uk = 2n − k при всех k. Рассмотрим теперь клетки, расположенные между k-й и (k + 1)-й линиями сетки. Хотя бы uk = 2n − k из этих клеток содержат по верхнему треугольнику, и хотя бы dk+1 = k + 1 из них содержат по нижнему. Значит, свободных клеток в этом ряду не больше, чем 2n−max(uk , dk+1), то есть не больше k при k < n и не больше (2n − 1) − k при k > n. Итого, общее число свободных клеток не больше, чем 2(0+1+. . .+(n−1)) = n(n−1). Осталось привести пример, на котором эта оценка достигается. Пример при n = 50 строится аналогично: выделяется «прямоугольник» из клеток со сторонами из n + 1 и n клеток, параллельными диагоналям доски, его клетки красятся в шахматном порядке (так, что угловые клетки прямоугольника — черные), и во все черные клетки кладется по два треугольничка (при этом n(n − 1) белых клеток остаются свободными); в оставшихся же четырех «углах» доски треугольнички кладутся так, что прямой угол ",2450,allrus,11 +"Назовем число интересным, если любые две его соседние цифры отличаются на 2. Сколько 14-значных интересных чисел делится на 11?","Воспользуемся критерием делимости на 11: «число делится на 11, если знакочередующаяся сумма S его цифр делится на 11». Разобьем цифры числа на 7 пар. Тогда в S 7 двоек. Но данная сумма четна и по модулю меньше 22. Поэтому интересное число может делиться на 11, только если S = 0 . Но это также невозможно, поскольку в S входит нечетное число двоек (и либо больше будет со знаком плюс, либо со знаком минус). Поэтому ни одно интересное число не может делится на 11.",0,mipt,11 +"Назовем число интересным, если любые две его соседние цифры отличаются на 2. Сколько 18-значных интересных чисел делится на 11?","Воспользуемся критерием делимости на 11: «число делится на 11, если знакочередующаяся сумма S его цифр делится на 11». Разобьем цифры числа на 9 пар. Тогда в S 9 двоек. Но данная сумма четна и по модулю меньше 22. Поэтому интересное число может делиться на 11, только если S = 0 . Но это также невозможно, поскольку в S входит нечетное число двоек (и либо больше будет со знаком плюс, либо со знаком минус). Поэтому ни одно интересное число не может делится на 11.",0,mipt,11 +"В одной тетради Вася записал 11 натуральных чисел. В другую тетрадь Петя записал наибольшие общие делители каждой пары чисел, записанных в васиной тетради. Оказалось, что каждое число, записанное в одной из двух тетрадей, есть и в другой тетради. Какое наибольшее количество различных чисел могло быть написано в васиной тетради?","Заметим вначале, что для любых натуральных чисел A >= B выполняется неравенство НОД(A, B) <= A, причем равенство выполняется только в случае, когда A = B. Пусть A >= B – два самых больших числа в васиной тетради. Тогда в петиной тетради число A может появиться только в одном случае, когда A = B ; НОД всех других пар чисел будет меньше A. Значит, в васиной тетради не больше 10 различных чисел. Если Вася запишет в тетрадь числа 2, 4, 8, 16, 32, 64, 128, 256, 512, 1024, 1024, то в петиной тетради будут записаны только такие же числа.",10,mipt,11 +"В комнате находятся несколько рыцарей и лжецов (рыцари всегда говорят правду, а лжецы всегда лгут). Каждому дали листок бумаги и попросили написать про каждого из остальных, кем он является — лжецом или рыцарем. Когда собрали все листы бумаги, оказалось, что всего записей «лжец» оказалось 48, а записей «рыцарь» — 42. Сколько в комнате лжецов, если известно, что их меньше рыцарей?","Пусть в комнате находятся x человек, тогда каждый из них написал x − 1 слово, поэтому x(x − 1) = 42 + 48 = 90, то есть x = 10. Пусть в комнате y лжецов, тогда рыцарей — (10 − y). Поэтому ответов «лжец» каждый из рыцарей дал y раз, а каждый из лжецов — 10 − y раз. Имеем: y(10 − y) + (10 − y)y = 48 следовательно y**2 − 10y + 24 = 0 следовательно y = 4 или y = 6. Но лжецов — меньше, поэтому y = 4.",4,mipt,11 +"В комнате находятся несколько рыцарей и лжецов (рыцари всегда говорят правду, а лжецы всегда лгут). Каждому дали листок бумаги и попросили написать про каждого из остальных, кем он является — лжецом или рыцарем. Когда собрали все листы бумаги, оказалось, что всего записей «лжец» оказалось 42, а записей «рыцарь» — 48. Сколько в комнате лжецов, если известно, что их меньше рыцарей?","Пусть в комнате находятся x человек, тогда каждый из них написал x − 1 слово, поэтому x(x − 1) = 48 + 42 = 90, то есть x = 10. Пусть в комнате y лжецов, тогда рыцарей — (10 − y). Поэтому ответов «лжец» каждый из рыцарей дал y раз, а каждый из лжецов — 10 − y раз. Имеем: y(10 − y) + (10 − y)y = 42 следовательно y**2 − 10y + 21 = 0 следовательно y = 3 или y = 7. Но лжецов — меньше, поэтому y = 3.",3,mipt,11 +"Найдите количество восьмизначных чисел, произведение цифр каждого из которых равно 16875. Ответ необходимо представить в виде целого числа.","Ввиду того, что 16875 = 3**3 * 5**4 , искомые числа могут состоять из следующих цифр: (а) три тройки, четыре пятёрки и одна единица или (б) тройка, девятка, четыре пятёрки и две единицы. Вычислим количество вариантов в каждом случае. (а) Сначала выбираем три места из восьми для расположения троек, затем одно место из пяти оставшихся для размещения единицы (5 способов). Наконец, оставшиеся места занимают пятёрки. По правилу произведения выходит 280 способов. (б) Рассуждая аналогично, находим, что количество способов в этом случае равно 840. Окончательно получаем 280 + 840 = 1120 способов.",1120,mipt,11 +"Найдите количество восьмизначных чисел, произведение цифр каждого из которых равно 9261. Ответ необходимо представить в виде целого числа.","Ввиду того, что 9261 = 3**3 * 7**3 , искомые числа могут состоять из следующих цифр: (а) три тройки, три семёрки и две единицы или (б) тройка, девятка, три семёрки и три единицы. Вычислим количество вариантов в каждом случае. (а) Сначала выбираем три места из восьми для расположения троек, затем три места из пяти оставшихся для размещения семёрок . Наконец, оставшиеся места занимают единицы. По правилу произведения выходит 560 способов. (б) Рассуждая аналогично, находим, что количество способов в этом случае равно 1120. Окончательно получаем 560 + 1120 = 1680 способов.",1680,mipt,11 +"Дан правильный 60-угольник. В нем провели все диагонали, соединяющие вершины, между которыми не более 19 других вершин. Сколько равнобедренных треугольников нарисовано? (Сторонами равнобедренного треугольника могут быть стороны и проведенные диагонали данного правильного многоугольника.)","Посчитаем основания равнобедренных треугольников, которые не содержат центр многоугольника. Треугольник будет равнобедренным, если между точками основания будет нечетное число отмеченных точек, то есть 19, 17, 15, …, 1. Отрезков каждого вида ровно 60. Поэтому таких треугольников 60 * 10 = 600. Также возможен случай, когда треугольник содержит центр многоугольника. Это возможно только если между вершинами лежит ровно по 19 точек, и такие треугольники",620,mipt,9 +"Дан правильный 90-угольник. В нем провели все диагонали, соединяющие вершины, между которыми не более 29 других вершин. Сколько равнобедренных треугольников нарисовано? (Сторонами равнобедренного треугольника могут быть стороны и проведенные диагонали данного правильного многоугольника.)","Посчитаем основания равнобедренных треугольников, которые не содержат центр многоугольника. Треугольник будет равнобедренным, если между точками основания будет нечетное число отмеченных точек, то есть 29, 27, 25, …, 1. Отрезков каждого вида ровно 90. Поэтому таких треугольников 90 * 15 = 1350. Также возможен случай, когда треугольник содержит центр многоугольника. Это возможно только если между вершинами лежит ровно по 29 точек, и такие треугольники будут равносторонними. Их количество равно 30. Итого получаем 1380 треугольников.",1380,mipt,9 +"Когда новый ученик Вася пришел в класс, средний балл по алгебре вырос на 0,04. Какое наибольшее число учеников могло быть в классе до прихода Васи, если в классе было меньше 30 учеников?","Пусть в классе было x учеников, их суммарный балл по алгебре равнялся S, а Васина оценка по алгебре A. Тогда (S + A) / (x + 1) - S/x = 1/25. Отсюда (Ax - S) / x*(x + 1), то есть x*(x + 1) = 25(Ax - S). Значит, x(x + 1) делится на 25 = 5**2. Числа x и x+1 взаимно простые, поэтому либо x, либо x+1 делится на 25. Значит, в классе было либо 24, либо 25 учеников. Покажем, что в классе могло быть 25 учеников. Пусть S = 99, A = 5, x = 25 , тогда (S + A) / (x + 1) - S/x = 104/26 - 99/25 = 1/25.",25,mipt,9 +"Когда новый ученик Вася пришел в класс, средний балл по алгебре вырос на 0,02. Какое наибольшее число учеников могло быть в классе до прихода Васи, если в классе было меньше 30 учеников?","Пусть в классе было x учеников, их суммарный балл по алгебре равнялся S, а Васина оценка по алгебре A. Тогда (S + A) / (x + 1) - S/x = 1/50. Отсюда (Ax - S) / x*(x + 1), то есть x*(x + 1) = 50(Ax - S). Значит, x(x + 1) делится на 25 = 5**2. Числа x и x+1 взаимно простые, поэтому либо x, либо x+1 делится на 25. Значит, в классе было либо 24, либо 25 учеников. Покажем, что в классе м��гло быть 25 учеников. Пусть S = 112, A = 5, x = 25 , тогда (S + A) / (x + 1) - S/x = 117/26 - 112/25 = 1/50.",25,mipt,9 +"На столе лежат 100 различных карточек с числами 3, 6, 9, . . . 297, 300 (на каждой карточке написано ровно одно число, каждое число встречается ровно один раз). Сколькими способами можно выбрать 2 карточки так, чтобы сумма чисел на выбранных карточках делилась на 5?","Данные числа, расположенные в порядке возрастания, образуют арифметическую прогрессию с разностью 3. Следовательно, остатки от деления на 5 у этих чисел чередуются. Действительно, если какое-то из этих чисел делится на 5, т.е. имеет вид 5𝑘, где 𝑘 принадлежит N, то следующее за ним число есть 5𝑘 + 3 – и оно даёт остаток 3 от деления на 5, далее – 5𝑘 + 6 = 5(𝑘 + 1) + 1, дающее остаток 1 от деления на 5, затем – 5𝑘 + 9 = 5(𝑘 + 1) + 4, дающее остаток 4 от деления на 5, затем 5𝑘 + 12 = 5(𝑘 + 2) + 2, дающее остаток 4 от деления на 5; наконец, следующим является 5𝑘 + 15 = 5(𝑘 + 3), которое снова делится на 5, после чего порядок остатков повторяется. Таким образом, остатки от деления данных чисел на 5 идут в порядке ... 0; 3; 1; 4; 2; 0 ... Среди данных нам 100 чисел есть по 20 чисел, дающих остатки 0, 1, 2, 3, 4 от деления на 5. Сумма двух чисел может делиться на 5 в следующих случаях. 1) Оба числа делятся на 5. Всего карточек с такими числами 20, и нужно выбрать 2 них – есть 1/2 * 20 * 19 = 190 способов сделать это. 2) Одно из чисел даёт остаток 1 от деления на 5 – тогда второе должно давать остаток 4 от деления на 5. Эту пару чисел можно выбрать 20 * 20 = 400 способами. 3) Одно из чисел даёт остаток 2 от деления на 5 – тогда второе даёт остаток 3, и, аналогично второму случаю, получаем 400 способов выбрать 2 числа. В итоге выходит 990 способов.",990,mipt,9 +"Даны две линейные функции 𝑓(𝑥) и 𝑔(𝑥) такие, что графики 𝑦 = 𝑓(𝑥) и 𝑦 = 𝑔(𝑥) – параллельные прямые, не параллельные осям координат. Найдите наименьшее значение функции (𝑔(𝑥))**2 + 𝑓(𝑥), если наименьшее значение функции (𝑓(𝑥))**2 + 𝑔(𝑥) равно −6. +","Пусть 𝑓(𝑥) = 𝑎𝑥 + 𝑏, 𝑔(𝑥) = 𝑎𝑥 + 𝑐, где 𝑎 != 0. Рассмотрим ℎ(𝑥) = (𝑓(𝑥))**2 + 𝑔(𝑥). Раскрывая скобки, получаем ℎ(𝑥) = (𝑎𝑥 + 𝑏)**2 + (𝑎𝑥 + 𝑐) = 𝑎**2𝑥**2 + 𝑎(2𝑏+1)𝑥 + 𝑏**2 + 𝑐. График 𝑦 = ℎ(𝑥) – это парабола с ветвями вверх, минимальное значение принимается в вершине. Абсциссой вершины является 𝑥в = −(2𝑏 + 1) / 2𝑎 ; ордината вершины равна ℎ (𝑥в) = −𝑏 − 1/4 + 𝑐. Аналогично получаем, что минимальное значение выражения (𝑔(𝑥))**2 + 𝑓(𝑥) равно −𝑐 − 1/4 + 𝑏. Заметим, что сумма этих двух минимальных значений равна − 1/2 , следовательно, если одно из этих минимальных значений равно −6, то второе равно − 1 2 + 6 = 11/2 = 5,5 .",5.5,mipt,9 +"На каждой из прямых 𝑦 = 3 и 𝑦 = 4 отмечено по 73 точки с абсциссами 1, 2, 3, ... , 73. Сколькими способами можно выбрать три точки из отмеченных 146 так, чтобы они являлись вершинами прямоугольного треугольника?","Есть две возможности. +1) Гипотенуза треугольника лежит на одной из прямых, а вершина прямого угла – на второй прямой. Пусть 𝐴𝐵𝐶 – данный треугольник с прямым углом при вершине 𝐶, 𝐶𝐻 – его высота, опущенная на гипотенузу. Из пропорциональности отрезков прямоугольного треугольника получаем, что 𝐶𝐻**2 = 𝐴𝐻 * 𝐵𝐻, т.е. 𝐴𝐻 * 𝐵𝐻 = 1. Поскольку 𝐴𝐻 и 𝐵𝐻 – целые числа, то 𝐴𝐻 = 𝐵𝐻 = 1. Гипотенузу 𝐴𝐵, равную 2, можно расположить 71 * 2 = 142 способами (по 73 − 2 способов расположения на каждой из двух данных прямых), при этом положение вершины 𝐶 определяется однозначно. 2) Один из катетов треугольника (назовём его 𝐵𝐶) перпендикулярен данным прямым, а второй катет (𝐴𝐶) лежит на одной из данных прямых. Тогда положение катета 𝐵𝐶 можно выбрать 73 способами. Для каждого варианта расположения катета 𝐵𝐶 вершину 𝐴 можно расположить 144 способами (подходят все точки кроме уже выбранных 𝐵 и 𝐶) – всего выходит 73 * 144 = 10512 способов. Итого получаем 142 + 10512 = 10654 способа.",10654,mipt,9 +"На координатной плоскости рассматриваются квадраты, все вершины которых имеют целые неотрицательные координаты, а центр находится в точке (60; 45). Найдите количество таких квадратов.","Проведём через данную точку (60; 45) вертикальную и горизонтальную прямые (𝑥 = 60 и 𝑦 = 45). Возможны два варианта. а) Вершины квадрата лежат на этих прямых (а его диагонали параллельны осям координат). Тогда “нижняя” вершина квадрата может быть расположена 45 способами: (60; 0),(60; 1), ... ,(60; 44) (положение остальных вершин при этом определяется однозначно). б) Вершины квадрата не лежат на указанных прямых. Это означает, что вершины лежат по одной в каждой из четырёх частей, на которые прямые 𝑥 = 60 и 𝑦 = 45 разделяют плоскость. Рассмотрим “левую нижнюю” вершину (её местоположение однозначно определяет остальные вершины). Для того, чтобы координаты всех вершин квадрата оказались неотрицательными, необходимо и достаточно, чтобы эта вершина попала в квадрат 15 <= 𝑥 <= 59, 0 <= 𝑦 <= 44. Получаем 45**2 способов. Общее количество способов равно 45**2 + 45 = 46 * 45 = 2070. ",2070,mipt,9 +"На координатной плоскости рассматриваются квадраты, все вершины которых имеют натуральные координаты, а центр находится в точке (55; 40). Найдите количество таких квадратов.","Проведём через данную точку (55; 40) вертикальную и горизонтальную прямые (𝑥 = 55 и 𝑦 = 40). Возможны два варианта. а) Вершины квадрата лежат на этих прямых (а его диагонали параллельны осям координат). Тогда “нижняя” вершина квадрата может быть расположена 39 способами: (55; 1),(55; 1), . . . ,(55; 39) (положение остальных вершин при этом определяется однозначно). б) Вершины квадрата не лежат на указанных прямых. Это означает, что вершины лежат по одной в каждой из четырёх частей, на которые прямые 𝑥 = 55 и 𝑦 = 40 разделяют плоскость. Рассмотрим “левую нижнюю” вершину (её местоположение однозначно определяет остальные вершины). Для того, чтобы координаты всех вершин квадрата оказались натуральными, необходимо и достаточно, чтобы эта вершина попала в квадрат 16 <= 𝑥 <= 54, 1 <= 𝑦 <= 39. Получаем 39**2 способов. Общее количество способов равно 39**2 + 39 = 39 * 40 = 1560.",1560,mipt,9 +"Известно, что для трёх последовательных натуральных значений аргумента квадратичная функция 𝑓(𝑥) принимает значения 13, 13 и 35 соответственно. Найдите наименьшее возможное значение 𝑓(𝑥).","Пусть 𝑛, 𝑛 + 1, 𝑛 + 2 – три данные последовательные значения аргумента. Поскольку квадратичная функция принимает одинаковые значения в точках, симметричных относительно абсциссы вершины параболы 𝑥в, то 𝑥в = 𝑛 + 0,5, а значит, 𝑓(𝑥) может быть представлена в виде 𝑓(𝑥) = 𝑎 (𝑥 − 𝑛 − 0,5)**2 + 𝑐. Так как 𝑓(𝑛) = 13, 𝑓(𝑛 + 2) = 35, то получаем 𝑎/4 + 𝑐 = 13, 9𝑎/4 + 𝑐 = 35, откуда 𝑎 = 11, 𝑐 = 41/4 . Но 𝑐 = 𝑓(𝑥в) и есть наименьшее значение функции.",4.25,mipt,9 +"На полуокружности равномерно отметили 200 точек (расстояние между любыми двумя соседними точками равны). Любые две точки, между которыми не более 19 точек, соединили отрезком. Сколько равнобедренных треугольников с вершинами в отмеченных точках нарисовано?","Посчитаем основания этих равнобедренных треугольников. Треугольник будет равнобедренным, если между точками основания будет нечетное число отмеченных точек. Если между точками основания лежит 19 точек, то таких оснований будет 200 - 19 - 1 = 180. Если 17 – то 182, если 15 – то 184, …, если 1 – то 198. Таким образом, искомое количество равно (180 + 198)/2 * 10 = 1890.",1890,mipt,10 +"На полуокружности равномерно отметили 210 точек (расстояние между любыми двумя соседними точками равны). Любые две точки, между которыми не более 17 точек, соединили отрезком. Сколько равнобедренных треугольников с вершинами в отмеченных точках нарисовано?","Посчитаем основания этих равнобедренных треугольников. Треугольник будет равнобедренным, если между точками основания будет нечетное число отмеченных точек. Если между точками основания лежит 17 точек, то таких оснований будет 210 - 17 - 1 = 192. Если 15 – то 194, если 13 – то 196, …, если 1 – то 208. Таким образом, искомое количество равно (192 + 208)/2 * 9 = 1800.",1800,mipt,10 +"На плоскости с заданной прямоугольной декартовой системой координат нарисован квадрат с вершинами в точках (0; 0), (0; 59), (59; 59) и (59; 0). Найдите количество способов выбрать два узла сетки внутри этого квадрата (не включая его границу) так, чтобы хотя бы один из этих узлов лежал на одной из прямых 𝑦 = 𝑥 или 𝑦 = 59 − 𝑥, но оба выбранных узла не лежали ни на какой прямой, параллельной любой из координатных осей.","Возможны два случая. 1) Оба выбранных узла лежат на указанных в условии прямых. На каждой из них внутри квадрата лежат по 58 точек, причём повторяющихся среди них нет (точка пересечения прямых имеет нецелые координаты). Для выбора первой точки есть 116 способов, а для выбора второй – на 3 меньше (подходят все точки, кроме первой и двух точек, лежащих с первой на той же горизонтали или вертикали). При этом мы учитывали упорядоченные пары точек, поэтому каждую пару точ��к мы посчитали дважды. Значит, в этом случае получаем 116 * 113/2 = 6554 способа. 2) Ровно один из выбранных узлов лежит на данных в условии прямых. Выберем сначала узел, лежащий на одной из прямых (116 способов). Посчитаем, сколько после этого есть способов выбрать второй узел. Всего в квадрате отмечены 58**2 узлов; из них мы должны исключить узлы на диагоналях (116 штук), а также узлы, стоящие с выбранным на одной горизонтали (56 штук с учётом исключённых ранее диагональных) или на одной вертикали (56 штук). Отсюда второй узел можно выбрать 582−116−112 = 3136 способами, а количество способов выбрать пару узлов равно 116 * 3136 = 363776. Подводя итоги, имеем 6554 + 363776 = 370330 способов.",370330,mipt,10 +"На столе лежит 55 кучек конфет. В одной кучке лежит 1 конфета, в другой — две, в третьей — 3, …, в последней — 55. Петя и Вася играют в следующую игру, делая ходы по очереди; начинает Петя. За один ход игрок берёт одну конфету из любой кучки. Если игрок забрал из кучки последнюю конфету, то он её съедает, а иначе выбрасывает. Игра продолжается до тех пор, пока все конфеты из кучек не будут съедены или выброшены. Какое наибольшее количество конфет может гарантированно съесть Петя?","Понятно, что Петя может съесть 1 конфету, например, если самым первым ходом заберёт конфету из кучки с 1 конфетой. Докажем, что Вася может помешать Пете съесть больше 1 конфеты. Для этого Вася будет действовать следующим образом. Если в какой-то кучке осталась ровно 1 конфета, он заберёт её и съест. Если же кучек с 1 конфетой нет, он будет брать конфету из любой кучки, в которой более 2 конфет. Для начала поймём, почему Вася всегда сможет сделать ход по такой стратегии. Предположим, что в какой-то момент Вася не может сделать ход, то есть в каждой кучке не больше 2 конфет, при этом нет кучек с 1 конфетой. Тогда в каждой кучке ровно 2 конфеты, и перед ходом Васи осталось чётное количество конфет. С другой стороны, изначально на столе конфет было 1 + 2 + … + 55 = 55 * 56/2 = 1540, т. е. чётное количество. Значит, после хода Пети должно оставаться нечётное количество конфет, а после хода Васи — чётное количество, противоречие. Теперь докажем, что при такой стратегии Васи Петя не сможет съесть больше 1 конфеты. Заметим, что если после какого-то хода Пети нет кучек из 1 конфеты, то их больше никогда и не будет. Действительно, кучки, из которых берёт конфеты Вася, после его хода не могут состоять только из 1 конфеты, а все кучки из 1 конфеты, которые оставляет Петя, Вася сразу же съедает следующим ходом. Таким образом, если Петя на первом ходу съест кучку из 1 конфеты, больше он конфет никогда не съест. Если же он не будет этого делать, её следующим ходом съест Вася, а Петя успеет за свой первый ход сделать не более одной новой кучки из 1 конфеты. Если она всё-таки появится (из кучки с 2 конфетами), и Петя не съест её на своём втором ходу, то он не съест вообще ничего, так как новой кучки из 1 конфеты на втором ходу он образовать не сможет. А если съест, то, как и ранее, больше ничего съесть не сможет. Итак, у Васи есть стратегия, позволяющая не дать Пете съесть более 1 конфеты",1,hse,8 +Вычислите сумму 1**2 + 2**2 − 3**2 − 4**2 + 5**2 + 6**2 − 7**2 − 8**2 + 9**2 + 10**2 − ... + 2017**2 + 2018**2,"Заметим, что при любом k верно равенство k**2 − (k + 1)**2 − (k + 2)**2 + (k + 3)**2 = 4. Поэтому вся сумма равна 1 + 504 * 4 + 2018**2 = 4074341.",4074341,hse,8 +"Сколькими способами из цифр 1, 2, 3, 4 можно составить число, кратное 6? При составлении числа каждую цифру можно использовать один раз или не использовать совсем","Число делится на 6, тогда и только тогда, когда оно делится на 2 и на 3. Число, составленное из цифр 1, 2, 3, 4 делится на 2, если и только если его последняя цифра чётная, то есть 2 или 4. Число делится на 3, тогда и только тогда, когда его сумма цифр делится на 3. В нашей ситуации такое возможно, для следующих наборов цифр: {1, 2}, или {1, 2, 3}, или {2, 4}, или {2, 3, 4} имеем 2 варианта в первом случае, 2 варианта во втором и 4 в последнем третьем случае. Итого 9 вариантов",9,hse,8 +"Найдите наименьшее натуральное число, которое можно получить при подстановке натуральных чисел вместо переменных в следующее выражение 13x**2 + y**2 + z**2 − 4xy − 6xz + y.","Заметим, что 13x**2 + y**2 + z**2 − 4xy − 6xz + y = (2x − y)**2 + (3x − z)**2 + y (1) Поскольку квадрат целого числа всегда неотрицательное число, он достигает минимума, когда равен 0. Натуральное число y не меньше 1. Если же y = 1, то число (2x − y) — нечётное и его квадрат также не меньше 1. Поэтому выражение (1) не меньше 2 для любых натуральных x, y, z. Значение 2 может быть достигнуто несколькими способами, например, x= 1,y = 2,z = 3 или x = 1, y = 1, z = 3. ",2,hse,8 +Какое максимальное количество полосок 5*1 можно вырезать из квадрата на клетчатой бумаге размера 8*8 клеток?,"Заметим, что больше 12 фигурок из 5 клеток в каждой поместить на клетчатую бумагу в которой всего 8 * 8 = 64 клетки заведомо не удастся (т. к. 64 = 12 * 5 + 4). Поэтому остается подыскать пример из 12 полосок.",12,hse,8 +"В стране из 2018 городов каждая пара городов соединена одной дорогой. Власти решили присвоить каждой трассе статус «федеральной» или «социальной», и для этой цели выпустили метки «Ф» и «С» суммарным числом, равным числу дорог. Однако рабочие расставили метки неправильно: на некоторых трассах могло оказаться по одной метке обоих видов, а на некоторых могло не оказаться ни одной. (Случай, когда на каждой дороге — ровно по одной метке, также считается возможным.) Каково максимально возможное число дорог с меткой «федеральная», если для любой такой дороги на каждой, не имеющей с ней общих концов, есть метка «социальная»?","Будем называть дорогу федеральной, если она имеет метку «Ф», даже если она при этом имеет метку «С». Если есть две федеральные дороги без общих концов (пусть это дороги А–Б и В–Г), то федеральных дорог не более 6 (потому что все дороги, кроме дорог между городами А, Б, В, Г, обязательно имеют метку «С», а число меток равно числу дорог). Если любые две федеральные дороги имеют общий конец, то рассмотрим две из них: А–Б и Б–В. Тогда либо есть ещё только одна федеральная дорога А–В (в таком случае федеральных дорог больше нет, т. е. их всего 3), либо все федеральные дороги имеют своим концом город Б (в таком случае федеральных дорог не более 2017). Случай с 2017 федеральными дорогами возможен (все дороги из одного города имеют метку «Ф», все остальные дороги – метку «С»).",2017,hse,8 +"Шесть почти честных пиратов закопали добытые золотые монеты на необитаемом острове и пустились в бега. Через год первый пират вернулся на остров, разделил все монеты на шесть равных частей, одна монета оказалась лишней. Пират забрал себе одну из частей и лишнюю монету, а остальное закопал. То же самое сделали по очереди остальные пираты, причем никто из них не знал о действиях других. Через много лет ученый археолог наткнулся на закопанные монеты. Какое наименьшее количество монет мог найти археолог?","Заметим, что после каждого перезакапывания число монет делится на 5. Пусть археолог нашёл n, монет, тогда n = 5a. Значит, шестой пират нашёл 6a + 1, что также делится на 5, то есть a = 4 % 5. Значит, a = 5b − 1, то есть 6a + 1 = 5(6b−1). Пятый нашёл 6(6b − 1) + 1 = 6**2b − 5. При этом 6**2b − 5 делится на 5, откуда b делится на 5, то есть b = 5c, 6**2b − 5 = 5(6c**2 − 1). Продолжая таким образом, получаем, что второй нашёл 6(6**4e − 1) + 1 = 6**5e − 5. При этом e делится на 5, то есть e = 5f, 6**5 e − 5 = 5(6**5f − 1). Первый пират нашёл 6**6f − 5, но это уже не имеет значения. Таким образом, b = 5c - ... = 5**4f. Тогда n=5a=5(5b−1)=5**6f −5. Поскольку f > 1, b > 5**6 − 5 = 15620.",15620,hse,8 +"Чётное число 2N >2 называется подходящим, если оно делится на модуль разницы между наибольшим из своих чётных делителей, отличных от 2N, и наибольшим из своих нечётных делителей. Сколько существует подходящих чётных чисел, не превосходящих 2018?","Предположим, что число 2N подходящее. Пусть 2N = 2**k*m, где m нечётное. Если k >= 2, то условие говорит, что 2**k*m делится на 2**(k−1)m − m = m(2**(k−1) −1), что возможно только при условии k = 2. Если k = 1 и m = ps, где p минимальный простой нечетный делитель m, то 2ps делится на 2s − ps = (2 − p)s, откуда имеем p−2p, значит p=3. Число N или имеет остаток 2 по модулю 4 или имеет остаток 3 по модулю 6. Тем самым число 2N является подходящим, если число N может иметь остаток 2, 3, 6, 9, 10 по модулю 12. Это значит, что в каждом ряду из 12 последовательных четных чисел ровно пять подходящих. Используя равенство 2018 = 2 * (12 * 84 + 1), получаем ответ 420 = 5 * 84.",420,hse,8 +"Действительные числа 𝑎, 𝑏, 𝑐, 𝑑 таковы, что 𝑎 + 𝑏 = 9/(𝑐 − 𝑑) и 𝑐 + 𝑑 = 25/(𝑎 − 𝑏). Какое наименьшее значение может принимать величина 𝑎**2 + 𝑏**2 + 𝑐**2 + 𝑑**2 ?","Если данные равенства домножить на знаменатели соответствующих дробей и сложить, мы получим 2(𝑎𝑐 − 𝑏𝑑) = 34. Докажем, что 𝑎**2 + 𝑏**2 + 𝑐**2 + 𝑑**2 >= 2(𝑎𝑐 − 𝑏𝑑). Это следует из 𝑎**2 + 𝑐**2 >= 2𝑎𝑐 (эквивалентно (𝑎 − 𝑐)**2 >= 0) и 𝑏**2 + 𝑑**2 >= −2𝑏𝑑 (эквивалентн�� (𝑏 + 𝑑)**2 >= 0). Значит, 𝑎**2 + 𝑏**2 + 𝑐**2 + 𝑑**2 >= 34. Равенство достигается, если все указанные выше неравенства обращаются в равенства, то есть при 𝑎 = 𝑐 и 𝑏 = −𝑑. Подставив эти соотношения в равенства, данные в условии, нетрудно найти подходящие значения, например 𝑎 = 4, 𝑏 = −1, 𝑐 = 4, 𝑑 = 1.",34,hse,9 +"Найдите все составные натуральные числа 𝑛, обладающие следующим свойством: каждый натуральный делитель числа 𝑛 (в частности, само 𝑛), уменьшенный на 1, является квадратом целого числа.","Предположим, что 𝑛 делится на квадрат какого-то простого числа 𝑝. Тогда у него есть делитель 𝑝**2 = 𝑏**2 +1; но два квадрата целых чисел могут отличаться на 1, только если это 0 и 1. Пусть 𝑛 делится на какие-то два простых числа 𝑝 и 𝑞. Без ограничения общности можно считать, что 𝑝 > 𝑞. Из условия, что любой делитель, уменьшенный на 1, является квадратом, можно записать 𝑝 = 𝑎**2 + 1, 𝑞 = 𝑏**2 + 1, 𝑝𝑞 = 𝑐**2 + 1. Вычтем из третьего уравнения первое, получим 𝑝𝑞 − 𝑝 = 𝑐**2 − 𝑎**2 . Это можно переписать в виде 𝑝(𝑞 − 1) = (𝑐 − 𝑎)(𝑐 + 𝑎). Так как 𝑝 — простое число, один из множителей в правой части должен делиться на 𝑝. Заметим, что из условия 𝑝 > 𝑞 следует, что 𝑝𝑞 < 𝑝**2 , откуда 𝑐 < 𝑝. Поэтому 𝑐 − 𝑎 < 𝑝 и, так как 𝑐 != 𝑎, не может делиться на 𝑝. Значит, 𝑐 + 𝑎 должно делиться на 𝑝. При этом 𝑐 < 𝑝 и 𝑎 < 𝑝, откуда 𝑐 + 𝑎 должно быть в точности равно 𝑝. Итак, получили, что 𝑐 = 𝑝 − 𝑎. Кроме того, так как 𝑝 = 𝑐 + 𝑎, 𝑞 − 1 должно быть равно оставшемуся множителю, т. е. 𝑐 − 𝑎. Значит, 𝑞 = 𝑐 − 𝑎 + 1 = 𝑝 − 2𝑎 + 1. Отсюда видно, что числа 𝑝 и 𝑞 разной чётности. Но так как они оба простые и 𝑝 > 𝑞, получаем, что 𝑞 = 2. Подставляя 𝑞 = 2, получаем 2 = 𝑐 − 𝑎 + 1 = 𝑝 − 2𝑎 + 1, откуда, во-первых, 𝑐 = 𝑎 + 1, а во-вторых, 𝑝 = 2𝑎 + 1. Тогда 𝑝𝑞 = 2𝑝 = 4𝑎 + 2 и 𝑝𝑞 = 𝑐**2 + 1 = (𝑎 + 1)**2 + 1. Приравнивая, получаем квадратное уравнение 𝑎**2 + 2𝑎 + 2 = 4𝑎 + 2, корнями которого являются числа 2 и 0, откуда 𝑝 равно 5 или 1. Но так как 𝑝 должно быть простым, то остаётся единственный вариант 𝑝 = 5. Таким образом, единственный возможный случай— это 𝑝 = 5, 𝑞 = 2. Понятно, что других простых чисел в разложении 𝑛 уже быть не может.",10,hse,10 +"Про вещественные числа a, b и c известно, что abc + a + b + c = 10, ab + bc + ac = 9. Для каких чисел x можно утверждать, что хотя бы одно из чисел a, b, c равно x? (Найдите все такие числа x и докажите, что других нет.)","Вычтем из первого равенства второе, преобразовав, получим (a − 1)(b − 1)(c − 1) = 0. Отсюда следует, что одно из a, b, c равно единице. Другие x не подходят, так как тройки (a, b, c) = (4, 1, 1) и (a, b, c) = (0, 9, 1) удовлетворяют условию.",1,hse,10 +"Рассмотрим всевозможные приведенные квадратные трёхчлены x**2 + px + q с целыми коэффициентами p и q. Назовём областью значений такого трехчлена множество его значений во всех целых точках x = 0, +1, -1, +2, -2 ... . Какое наибольшее количество таких трехчленов можно выбрать, чтобы их области значений попарно не пересекались?","Заметим, что замена переменной x на x + k при любом целом k не меняет области значений многочлена. Тогда, сделав замену x на x − [p/2] (квадратные скобки означают целую часть) можем считать, что любой многочлен имеет один из двух видов: x**2 + q или x**2 + x + q. Области значений любых двух многочленов разного вида пересекаются: в самом деле, значения многочленов x**2 + q и x**2 + x + q' совпадают при x = q − q'. Значит, многочлены разного вида брать нельзя. Многочленов первого вида можно выбрать не больше двух, поскольку если области значений f1(x) = x**2 + q и f2(x) = x**2+q' не пересекаются, то q−q' = 4k + 2 при некотором k принадлежащем Z. В самом деле, для нечетной разности свободных членов q − q' = 2k + 1 имеем f1(k) = f2(k + 1). Для делящейся на 4 разности свободных членов q − q' = 4k имеем f1(k − 1) = f2(k + 1). Но если выбрано хотя бы три многочлена, то среди попарных разностей свободных членов хотя бы одна не имеет вид 4k + 2. Многочленов второго вида тоже можно выбрать не больше двух, поскольку если области значений f1(x) = x**2 + x + q и f2(x) = x**2 + x + q' не пересекаются, то q − q' = 2k + 1 при некотором k принадлежащем Z. В самом деле, для четной разности свободных членов q − q' = 2k имеем f1(k − 1) = f2(k). Опять же, если выбрано хотя бы три многочлена, то среди попарных разностей свободных членов хотя бы одна четна. Итак, больше двух многочленов выбрать нельзя. Пример для двух: f1(x) = x**2 и f2(x) = x**2 + 2.",2,hse,10 +"Натуральные числа 𝑎, 𝑏, 𝑐 таковы, что 1 <= 𝑎 < 𝑏 < 𝑐 <= 3000. Найдите наибольшее возможное значение величины +НОД(𝑎, 𝑏) + НОД(𝑏, 𝑐) + НОД(𝑐, 𝑎).","Заметим, что НОД(𝑎, 𝑏) = НОД(𝑎, 𝑏 − 𝑎) <= 𝑏 − 𝑎, так как НОД двух натуральных чисел не превосходит каждое из них. Аналогично получаем, что НОД(𝑏, 𝑐) <= 𝑐−𝑏, а также НОД(𝑐, 𝑎) <= 𝑎. Складывая эти три неравенства, получаем НОД(𝑎, 𝑏) + НОД(𝑏, 𝑐) + НОД(𝑐, 𝑎) <= (𝑏 − 𝑎) + (𝑐 − 𝑏) + 𝑎 = 𝑐 <= 3000. В качестве примера на 3000 можно предъявить, например, 𝑎 = 1000, 𝑏 = 2000, 𝑐 = 3000. В этом случае НОД(𝑎, 𝑏) + НОД(𝑏, 𝑐) + НОД(𝑐, 𝑎) = 1000 + 1000 + 1000 = 3000.",3000,hse,11 +"Болельщики должны выбрать 6 лучших хоккеистов чемпионата: одного вратаря, двух защитников и трех нападающих. Среди претендентов: 3 вратаря, 5 защитников, 6 нападающих и 3 «универсала». «Универсал» — игрок, хороший в разных ролях, который поэтому может быть выбран как в качестве защитника, так в качестве нападающего (но не вратаря). Сколько существует способов выбрать эту шестерку? Требуется получить числовое значение.","С выбором вратаря проблем нет: 3 способа. При выборе защитника есть 3 возможности: а) оба защитника выбираются из 5-ти защитников: (5 * 4)/2 = 10; тогда при выборе нападающих есть 6 + 3 = 9 претендентов; б) один защитник выбирается из 5-ти защитников, а второй из 3-х «универсалов»; тогда при выборе нападающих есть 6 + 2 = 8 претендентов; в) оба защитника выбираются из 3-х «универсалов»; тогда при выборе нападающих есть 6 + 1 = 7 претендентов. Таким образом, общее количество вариантов равно: 3 * ( (5 * 4)/2 * (9 * 8 * 7)/(2 * 3) + 5 * 3 * (8 * 7 * 6)/(2 * 3) + (3 * 2)/2 * (7 * 6 * 5)/(2 * 3) ) = 3 * 7 * 5 * (3 * 8 + 8 * 3 + 3) = 105 * (24 + 24 + 3) = 5355.",5355,msu,11 +"В магазине продают три вида ручек: по 14 рублей, по 15 рублей и по 16 рублей. Вася купил ручек ровно на 170 рублей. Сколькими способами это можно было сделать?","Пусть Вася купил k ручек по 14 рублей, l ручек по 15 рублей и m ручек по 16 рублей. Тогда 14k + 15l + 16m = 170; для ответа на вопрос задачи нужно найти количество решений этого уравнения в неотрицательных целых числах. Перейдя в уравнении к остаткам от деления на 15, получим, что m = k + 5 + 15s, где s — некоторое целое число. Подставив это выражение в исходное уравнение и поделив обе его части на 15, получим 2k + l + 16s = 6. Так как 2k + l >= 0, то s <= 0; если s <= −2, то 2k + l >= 6 + 32 = 38 и 170 = 14k + 15l + 16m >= 14k + 7l >= 7 * 38 > 170 — противоречие, следовательно, s = 0 или s = −1. При s = 0 получаем k = 0 и l = 6, k = 1 и l = 4, k = 2 и l = 2, k = 3 и l = 0, при этом m = k + 5 равно 5, 6, 7, 8 соответственно. При s = −1 имеем m = k + 5 − 15 = k − 10 >= 0, значит, 2k >= 20 и 22 = 22 + 0 = 20 + 2, получаем k = 11 и l = 0, k = 10 и l = 2, при этом m равно 1 и 0 соответственно. Итого 6 вариантов.",6,msu,11 +"Даны два натуральных числа. Большее из них равно квадрату их разности, а меньшее из них в 8 раз больше их наибольшего общего делителя. Найдите наименьшее общее кратное этих двух чисел.","Пусть даны числа x <= y. По условию y = (y−x)**2 , откуда y = n**2 и x = n**2 − n, где n — некоторое натуральное число. Поскольку НОД(n**2 , n**2 − n) = НОД(n**2 , n) = n, из второго условия задачи получим x = 8n следовательно n**2 − n = 8n следовательно n = 9. Значит, x = 72, y = 81, НОК(72,81) = 648.",648,msu,11 +"Четыре купца Арсений, Богдан, Вакула и Гаврила получили из казны 50 золотых червонцев. Они их решили разделить так, чтобы каждому досталось нечётное количество червонцев. Сколько есть разных способов дележа?","Ответ на вопрос задачи равен количеству решений уравнения x1 + x2 + x3 + x4 = 50 в положительных нечётных числах. Пусть xi = 2yi −1 (i = 1, 2, 3, 4). Тогда все yi — натуральные числа, и уравнение приобретает вид y1 + y2 + y3 + y4 = 27. Количество решений этого уравнения в натуральных числах равно числу способов расставить в ряду из 27 шариков 3 перегородки (перегородки можно ставить только между двумя шариками, нельзя ставить две перегородки рядом): число y1 будет равно количеству шариков левее первой перегородки, y2 — между первой и второй, y3 — между второй и третьей, y4 — правее третьей перегородки. А число способов расстановки перегородок равно (26 * 25 * 24) / (2 * 3) = 2600.",2600,msu,11 +"Шайка пиратов нашла клад в 15000 золотых монет. Они договорились, что некоторые из них получат по 48 монет, а остальные — по 49 монет. Клад удалось поделить без остатка. Какое наименьшее число пиратов может получить по 49 монет?","Пусть x пиратов получили по 48 монет, а y — по 49. Тогда 48x + 49y = 15000, (50 − 2)x + (50 − 1)y = 15000, 50(x + y) − 2x − y = 15000, ((x + y) − 2x + y)/50 = 300. Таким образом, число 2x + y нацело делится на 50. Пусть 2x + y = 50k, где k — некоторое натуральное число. Тогда 2x + y = 50k, x + y = 300 + k. Если вычесть из первого уравнения второе, а из удвоенного второго — первое, получим x = 49k − 300, y = 600 − 48k. Таким образом, каждому натуральному k соответствуют единственно возможные значения x и y. Число x будет неотрицательным при k >= 7, а число y — при k <= 12. Значит, k принадлежит [7; 12]. При k = 7 получаем наименьшее значение x = 43 и наибольшее значение y = 264, а при k = 12 — наоборот, наибольшее значение x = 288 и наименьшее значение y = 24.",24,msu,11 +Сколько целочисленных решений у уравнения x**2 + (y**2 − 2022**2)**2 = sin(2023(x**3 − 1)) + 1?,"При подстановке в уравнение целочисленного решения левая часть уравнения принимает целочисленное значение. Значит, правая часть тоже принимает целочисленное значение, то есть синус может принимать значения −1, 0, 1. Синус не может принимать значения 1, −1 на целочисленном аргументе, поэтому он принимает значение 0, откуда с учётом целочисленности x получаем, что 2023(x**3 − 1) = 0 и x = 1. Отсюда находим, что y = 2022, -2022 то есть уравнение имеет два целочисленных решения",2,msu,11 +"Три различных корня уравнения 8x**3 − 12x**2 − 2x + a = 0 составляют арифметическую прогрессию. Найдите наибольший из корней, ответ при необходимости округлите до сотых","Пусть x1 < x2 < x3 — корни уравнения. Так как они образуют арифметическую прогрессию, то x2 = (x1+x3)/2 , поэтому x1 + x2 + x3 = 3 * x2. С другой стороны, по теореме Виета для многочлена третьей степени x1 + x2 + x3 = 12/8 = 3/2 , откуда x2 = 1/2 . Так как x2 = 0.5 — корень, то 8(0.5)**3 − 12(0.5)**2 − 2(0.5) + a = 0, откуда a = 3. Далее, 8x**3 − 12x**2 − 2x + 3 = (x − 0.5)(8x**2 − 8x − 6), и два оставшихся корня равны −0.5 и 1.5. Значит, наибольший корень равен 1.5",1.5,msu,11 +Обозначим через s(n) число цифр в десятичной записи натурального числа n. Найдите сумму s(2**2023) + s(5**2023),"Заметим, что s(2**2023) = lg(2**2023) + a = 2023lg2 + a, где 0 < a < 1. Аналогично, s(5**2023) = lg(5**2023) + b = 2023lg5 + b, где 0 < b < 1. Тогда s(2**2023) + s(5**2023) = 2023(lg2 + lg5) + a + b = 2023 + (a + b). Значит, a + b целое, причем 0 < a + b < 2, так как 0 < a < 1, 0 < b < 1. Отсюда a + b = 1, а ответ равен 2024. В общем случае — s(2**n) + s(5**n) = n + 1",2024,msu,11 +"Монета искривлена так, что вероятность выпадения ровно 3-х орлов в серии из 5- ти бросков равна вероятности выпадения ровно 2-х орлов в серии из 4-х бросков. Найдите вероятность того, что в серии из 6-ти бросков выпадет не менее 3-х орлов. Если необходимо, ответ округлите до сотых.","Пусть вероятность выпадения орла при одном броске равна p. Тогда вероятность выпадения ровно 3-х орлов в серии из 5-ти бросков равна 10p**3 * (1 − p)**2 , а вероятность выпадения ровно двух орлов в серии 4-х бросков равна 6p**2 * (1 − p)**2 . По условию 10p**3 * (1 − p)**2 = 6p**2 * (1 − p)**2 , откуда p = 3/5 . Тогда вероятность того, что в серии 6 бросков выпадет не менее 3-х орлов, равна 513/625 = 0.82. В варианте 2) p такая же, а вероятность выкинуть не менее 4 орлов за 6 бросков равна 7 * 3**5/5**5 = 1701/3125 = 0.54",0.54,msu,11 +"Числа a, b, c таковы, что каждое из двух уравнений x**2 + bx + a = 0 и x**2 + cx + a = 1 имеет по два целых корня, при этом все эти корни меньше (−1). Найдите наименьшее значение a.","По теореме Виета произведение корней первого уравнения равно a, произведение корней второго уравн��ния равно a − 1. Ввиду того, что корни целые и меньше −1, их произведение больше 1, поэтому каждое из двух последовательных чисел a − 1 и a является произведением двух различных целых чисел, больших 1. Так как первое нечетное число, не являющееся простым или квадратом простого, это 15, то получается a − 1 = 14, a = 15. Тогда корни первого уравнения −3 и −5 (при этом b = 8), корни второго уравнения −2 и −7 (при этом c = 9).",15,msu,11 +"Рассматриваются всевозможные наборы, которые состоят из 2017 различных натуральных чисел и в каждом из которых ни одно из чисел нельзя представить в виде суммы двух других чисел этого набора. Какое наименьшее значение может принимать наибольшее число в таком наборе? +","Положим для краткости n = 2017. Рассмотрим следующий набор из n чисел: n − 1, n, n + 1, . . ., 2n − 3, 2n − 2. Так как (n − 1) + n = 2n − 1 > 2n − 2, то ни одно из чисел этого набора не может равняться сумме двух других, то есть представленный набор удовлетворяет условию задачи. Пусть теперь имеется произвольный набор из n натуральных чисел, удовлетворяющий условию задачи. Докажем, что наибольшее из этих чисел N не меньше, чем 2n − 2. Пусть N <= 2n − 3. Докажем, что в таком наборе чисел есть пара чисел, меньших N, сумма которых равна N. Разобьём все натуральные числа, меньшие N, на пары с суммой, равной N: (1, N − 1); (2, N − 2), . . .. Если таких пар n − 1 или больше, то чисел, меньших N, будет не меньше, чем 2n − 2, что невозможно. Поэтому таких пар не больше, чем n − 2, а чисел в наборе n. Значит, в наборе найдётся, по крайней мере, одна пара чисел с суммой, равной N. Таким образом, в произвольном наборе из n чисел, удовлетворяющих условию задачи, наибольшее из этих чисел не меньше, чем 2n − 2 (причём эта оценка достижима). В случае n = 2017 это число 4032.",4032,msu,11 +"На доске было написано 21 последовательное натуральное число. Когда одно из чисел стерли, сумма оставшихся стала равна 2017. Какое число стерли?","Пусть на доске были написаны числа N-10, N-9,…,N, …, N+10. Их сумма равна 21N. Когда стерли одно их этих чисел – х - сумма стала равна 2017, 21Nx = 2017. Следовательно, x = 21N - 2017 , поскольку это одно из этих чисел, получаем 𝑁 − 10 <= 21𝑁 − 2017 <= 𝑁 + 10. Решим неравенства 2007/20 <= 𝑁 <= 2027/20 , откуда N =101, следовательно, x=21 * 101 - 2017 = 104.",104,msu,9 +Прямая проходит через точку с координатами (10; 0) и пересекает параболу 𝑦 = 𝑥**2 в точках с абсциссами 𝑥1 и 𝑥2. Найдите 1/𝑥1 + 1/𝑥2 .,"Точки x1,x2, являются решениями уравнения 𝑥**2 = 𝑘(𝑥 − 10), где k – коэффициент наклона прямой. Тогда по теореме Виета 𝑥1 + 𝑥2 = 𝑘, 𝑥1𝑥2 = 10𝑘. Следовательно, 1/𝑥1 + 1/𝑥2 = (𝑥1+𝑥2)/𝑥1𝑥2 = 𝑘/10𝑘 = 1/10 .",0.1,msu,9 +Сколько диагоналей в правильном 32-угольнике не параллельны ни одной из сторон этого 32-угольника?,"Всего в 32-угольнике 32 * (32 - 3)/2 = 464 диагоналей. Разобьем стороны на 16 пар параллельных сторон. Несложно заметить, что если зафиксировать какую-то пару, т.е. 4 вершины, то оставшиеся вершины можно соединить попарно диагоналями, параллельными этой паре. Их всего будет (32 - 4)/2 = 14. Значит диагоналей, параллельных какой-то стороне – 14 * 16 = 224. А не параллельных – 464 - 224 = 240.",240,msu,9 +"Про натуральные числа m и n известно, что 3𝑛**3 = 5𝑚**2 . Найдите наименьшее возможное значение m+n.","Очевидно, если m,n содержат простые сомножители, не равные 3 или 5, то на них можно сократить (и уменьшить m+n). Пусть 𝑛 = 3**𝑎 * 5**𝑏 , 𝑚 = 3**𝑐 * 5**𝑑 . Тогда из условия вытекает, что 3a + 1 = 2c, 3b = 2d + 1. Наименьшие возможные значения: a = 1, b = 1, c = 2, d = 1, откуда n = 15, m = 45.",60,msu,9 +"А у нас сегодня кошка родила вчера котят! Известно, что два самых легких весят в сумме 80 г., четыре самых тяжелых – 200 г., а суммарный вес всех котят равен X г. При каком наименьшем X по этим данным нельзя однозначно определить, сколько котят родила кошка?","Упорядочим котят по весу: 𝑎1 <= 𝑎2 <= ⋯ <= 𝑎𝑛. По условию 𝑎1 + 𝑎2 = 80, следовательно, 𝑎3, … , 𝑎𝑛 >= 40 . Так же по условию 𝑎(𝑛−3) + 𝑎(𝑛−2) + 𝑎(𝑛−1) + 𝑎𝑛 = 200, следовательно, 𝑎1, … , 𝑎(𝑛−4) <= 50. Получаем, что a3+. . . +a(n−4) = X − 280 = X1, при этом каждое слагаемое в левой части принадлежит отрезку [ 40; 50]. Значит 40 * (𝑛 − 6) <= 𝑋1 <= 50 * (𝑛 − 6), откуда вытекает, что n-6 лежит на [X1/50; X1/40]. Длина этого отрезка равна X1/200, значит два различных целых числа могут попасть только при 𝑋1 >= 200. Берем X1 = 200 и проверяем, что такие наборы весов существуют: 40 * 7 + 50 * 4 = 480 и 40 * 2 + 50 * 8 = 480. Для полноты решения надо добавить, что если X=200, то котят 4, если 200 < X < 280, то котят 5 и при X = 280 котят 6, т.е. количество котят тоже определяется однозначно",480,msu,8 +"Найдите двузначное число, цифры которого различны и квадрат которого равен кубу суммы его цифр","Запишем условие в виде 𝐴𝐵**2 = (𝐴 + 𝐵)**3 . Заметим, что сумма цифр (A + B) не превосходит 17, т.е. 𝐴𝐵̅**2 <= 17**3 . Кроме того, это число 𝐴𝐵̅**2 = 𝑛**6 , где n - некоторое натуральное число, которое не превосходит 4. Но 1, 2 не подходят, т.к. их кубы однозначны. Осталось 3 и 4, непосредственная проверка показывает, что 27**2 = (2 + 7)**2 = 729.",27,msu,8 +"На окружности отмечено 100 точек, которые покрашены в красный или синий цвет. Некоторые точки соединены отрезками, причем у любого отрезка один конец синий, а другой – красный. Известно, что не существует двух красных точек, принадлежащих одинаковому количеству отрезков. Каково наибольшее возможное число красных точек?","Возьмем 50 красных и 50 синих точек. Первую красную точку не соединяем ни с какой другой, вторую с одной синей, …, 50-ю – с 49 синими. Очевидно, что больше 50 красных точек быть не может, т.к. если их 51 или больше, то синих не более 49, следовательно, количество вариантов соединения не более 50, т.е. (по принципу Дирихле) какие-то две красные точки будут принадлежать одинаковому количеству отрезков.",50,msu,8 +"Бизнесмены Иванов, Петров и Сидоров решили создать автопредприятие. Иванов купил для предприятия 70 одинаковых автомобилей, Петров – 40 таких же автомобилей, а Сидоров внес в предприятие 44 миллиона рублей. Известно, что Иванов и Петров могут п��делить эти деньги между собой так, что вклад в общее дело каждого из трех бизнесменов будет одинаковым. Сколько денег полагается Иванову? Ответ дать в миллионах рублей.","1 способ. Каждый из бизнесменов должен внести столько же, сколько Сидоров, то есть 44 млн. руб. Если автомобиль стоит x, то (70x + 40x)/3 = 44 , x = 1.2 . Получается, что Иванов внес 70 * 1.2 = 84 млн. руб., поэтому он должен получить обратно 84 - 44 = 40 млн. руб. Петров внес 40 * 1.2 = 48млн. руб., поэтому он должен получить обратно 48 - 44 = 4 млн. руб. 2 способ. Каждый из бизнесменов должен внести столько же, сколько Сидоров, то есть 44 млн. руб. Если Иванов заберет t млн. руб., то Петрову останется ( 44 - t ) млн. руб. Поэтому 70x - t = 44 и 40x - (44 - t) = 44 (здесь x – цена автомобиля). Решая эту систему, получим x = 1.2 и t = 40.",40,msu,11 +"Сколько 9-значных чисел, делящихся на 5, можно составить путем перестановки цифр числа 377353752?","Так как число делится на 5, то на 9-м месте может стоять только пятерка. После этого нужно на оставшиеся 8 мест распределить 8 цифр: 3 семѐрки, 3 тройки, пятерку и двойку. Всего перестановок будет 8! , но так как есть повторяющиеся цифры, то ответ будет: 8! / 3! * 3! = (2 * 3 * 4 * 5 * 6 * 7 * 8) / (2 * 3 * 2 * 3) = 4 * 5 * 7 * 8 = 1120",1120,msu,11 +Найдите сумму всех корней уравнения x**2 - 31x + 220 = 2**x(31 - 2x - 2**x),"Исходное уравнение равносильно уравнению (x + 2**x)**2 - 31(x + 2**x) + 220 = 0 следовательно x + 2**x = 11, x + 2**x = 20. Каждое из уравнений этой совокупности имеет не более одного корня, так функция f(x) = x + 2**x – монотонно возрастающая. Первое уравнение имеет корень x = 3 , а второе – корень x = 4 . Сумма корней равна 7.",7,msu,11 +"Среди всех простых дробей, числитель и знаменатель которых являются двузначными числами, найдите наименьшую дробь, большую чем 3/4 . В ответе укажите ее числитель.","Требуется найти такую дробь a/b , при которой a/b - 3/4 = (4a - 3b)/4b достигает минимума. Поэтому ищется максимальное двузначное b, при котором 4a - 3b = 1. Если при этом получается b >= 50 , то дробь a/b - 3/4 = 1/4b будет всегда меньше, чем любая другая дробь с большим целым числителем и другим двузначным b. Решаем уравнение 4a - 3b = 1. Так как b = (4a - 1)/3 = a + (a - 1)/3 – целое, то a = 1 + 3k , где k – +произвольное целое число. Поэтому b = 1 +4k. Максимальным k, при котором a и b двузначные, +будет .k = 24 Поэтому b = 97 и a = 73, то есть искомая дробь: a/b = 73/97",73,msu,11 +"Средняя оценка на ЕГЭ по математике всех выпускников школы, в которой два выпускных класса, оказалась равна 58 баллам. При этом средняя оценка у учеников 11 «А» класса составляет 54.5 балла, а у учеников 11 «Б» класса – 62 балла. Сколько учеников в 11 «Б» классе, если известно, что по нормативам в классе должно быть не меньше 22 и не больше 39 учеников?","Пусть в 11 «А» классе x учеников, а в 11 «Б» классе – y учеников. Тогда (54.5 * x + 62 * y)/(x + y) = 58 , откуда получается 7x = 8y . Значит, x = 8n , y = 7n, где n принадлежит N. По условию 22 <= 8n <= 39 (отсюда n = 3 или 4), 22 <= 7n <= 39 (отсюда n = 4 или 5), поэтому n = 4 . Значит, в классах соответственно 32 и 28 учеников.",28,msu,11 +"Найдите все четырехзначн��е числа, которые при делении на 71 дают в остатке 27, а при делении на 79 дают в остатке 40. В ответ запишите сумму всех таких чисел.","Из условия задачи следует, что искомое число равно x = 71m + 27 или x = 79n + 40 . Отсюда получается уравнение в целых числах 71m + 27 = 79n + 40, или 71m - 79n = 13. Решение последнего уравнения (его можно найти разными путями): m = 28 + 79k , n = 25 + 71k , где k – произвольное целое число. Поэтому искомые числа: x = 2015 + 5609k. Из них четырѐхзначными являются: 2015 и 7624. Их сумма: 9639.",9639,msu,11 +"В ящике лежат сто разноцветных шариков: 28 красных, 20 зеленых, 13 желтых, 19 синих, 11 белых и 9 черных. Какое наименьшее число шариков надо вытащить, не заглядывая в ящик, чтобы среди них заведомо оказалось не менее 15 шариков одного цвета?","Наихудший вариант: будет вытащено 14 красных, 14 зеленых, 13 желтых, 14 синих, 11 белых и 9 черных шариков – всего 75 шариков. Следующий шарик обязательно будет 15-м шариком какого-то одного из цветов: либо красным, либо зеленым, либо синим.",76,msu,11 +"Шариковая ручка стоит 10 рублей, гелевая – 50 рублей, а перьевая – 80 рублей. Какое наибольшее количество гелевых ручек можно купить при условии, что всего нужно купить ровно 20 ручек и среди них должны быть ручки всех трех типов, а истратить на них нужно ровно 1000 рублей?","Если куплено x шариковых ручек, y – гелевых и z – перьевых, то имеем два уравнения: x + y + z = 20 ; 10x + 50y + 80z = 1000 (или x + 5y + 8z = 100). Вычитая из второго уравнения первое, получим 4y + 7z = 80. Отсюда следует, что z должно делиться на 4, т. е. z = 4n. Значит, 4y + 7 * 4n = 80, т. е. y = 20 - 7n. Соответственно x = 20 - y - z =3n. Положительные решения получаются при n = 1 и n = 2 . В итоге решением являются две тройки целых чисел: (3, 13, 4) и (6, 6, 8). Наибольшее возможное y равно 13.",13,msu,11 +"Найдите наименьшее значение a, при котором сумма квадратов корней уравнения x**2 -3ax + a**2 = 0 равна 0.28.","По теореме Виета x1**2 + x2**2 = (x1 + x2)**2 - 2x1x2 = (3a)**2 - 2a**2 = 7a**2. По условию 7a**2 = 28/100, отсюда a**2 = 1/25 и a = 0.2, -0.2 . Важно при этом, что при a = 0.2, -0.2 дискриминант D = 9a**2 - 4a**2 = 5a**2 > 0.",-0.2,msu,11 +"Первоклассница Маша, заходя в школу, каждый раз поднимается на школьное крыльцо по лестнице, имеющей 10 ступенек. Находясь внизу лестницы или на очередной ее ступеньке, она может либо подняться на следующую ступеньку, либо перепрыгнуть через одну ступеньку вверх (перепрыгнуть через две или более ступенек Маша пока не может). Какое минимальное количество раз Маше нужно зайти в школу, чтобы подняться на крыльцо всеми возможными способами?","Заметим, что на крыльцо из одной ступеньки Маша может подняться одним способом, а на крыльцо из двух ступенек – двумя: либо наступив на каждую ступеньку, либо, перешагнув через первую ступеньку, попасть сразу на вторую. Пусть n a – количество способов, которыми Маша может подняться на крыльцо, имеющее n ступенек. Так как на n-ю ступеньку Маша может подняться либо с (n - 1)-й ступеньки, либо с (n - 2) -й ступеньки, то an = a(n - 2) + a(n - 1). Последовательно вычисляем: a1 = 1, a2 = 2, a3 = 1 + 2 = 3, a4 = 2 + 3 = 5, a5 = 3 + 5 = 8, a6 = 5 + 8 = 13, a7 = 8 + 13 = 21, a8 = 13 + 21 = 34, a9 = 21 + 34 = 55, a10 = 34 + 55 = 89. Заметим, что числа, построенные по правилу F0 = 0, F1 = 1, F(n) = F(n - 2) + F(n - 1) (n >= 2), называются числами Фибоначчи. Таким образом, a(n) = F(n + 1).",89,msu,11 +"Сколько страниц в книжке, если для полной нумерации ее страниц (от первой до последней) потребовалось 708 цифр?","Однозначными и двузначными числами занумеровано 99 страниц – на это ушло 9 * 1 + 90 * 2 = 189 цифр. Значит, осталось 708 - 189 = 519 цифр, которыми записано 519/3 = 173 трехзначных числа.",272,msu,11 +"Из сосуда, до краев наполненного вкусным 100%-м соком, пятиклассница Маша за день отпила 1 л сока, а вечером долила в сосуд 1 л воды. На следующий день после тщательного перемешивания она выпила 1 л смеси и вечером долила 1 л воды. На третий день, снова перемешав смесь, она выпила 1 л этой смеси и вечером долила 1 л воды. Утром следующего дня родители выяснили, что объем воды в сосуде на 1,5 л больше объема оставшегося сока. Сколько литров сока выпила в итоге Маша? Если ответ на вопрос задачи неоднозначен, укажите сумму всех возможных значений искомой величины.","Пусть объем сосуда в литрах равен x. После первого дня в сосуде останется (x - 1) литр сока, после второго дня – (x - 1)**2 / x литров сока, а после третьего дня – (x - 1)**3 / x литров сока. Согласно условию получаем уравнение x - (x - 1)**2 / x = (x - 1)**3 / x + 1.5 следовательно x**3 - (x - 1)**3 = (x - 1)**3 + 3/2x**2 следовательно (x - 2)**2(x - 1/2) = 0. По условию x > 1 , следовательно x = 2 (это объем сосуда), сока осталось (2 - 1)**3/2**2 = 1/4литра. Значит, Маша выпила 2 - 0.25 = 1.75 литра.",1.75,msu,11 +"Среди чисел, превышающих 2013, найдите наименьшее четное число N, при котором дробь (15N - 7)/(22N - 5) сократима.","Наличие общего множителя у чисел 15N - 7 и 22N - 5 влечет за собой наличие такого же множителя у числа (22N - 5) - (15N - 7) = 7N + 2, а далее последовательно у чисел (15N - 7) - 2 * (7N + 2) = N - 11, (7N + 2) - 7 * (N - 11) = 79. Так как 79 – простое число, то дробь сократима на 79, поэтому N - 11 = 79m, N = 11 + 79m. По условию N – четное, поэтому N = 90 + 158p. Нужное значение достигается при p = 13.",2144,msu,11 +"Маша задумала 10-значное число и сообщила Васе, что остаток от деления этого числа на 9 равен 3. Потом Маша зачеркнула одну цифру и сказала Васе, что остаток от деления на 9 получившегося 9-значного числа равен 7. Помогите Васе угадать цифру, которую зачеркнула Маша. Запишите эту цифру в ответ.","По признаку делимости на 9 остаток от деления суммы цифр числа на 9 равен 3 (поэтому сумма цифр этого числа равна 9n + 3), а остаток от деления суммы цифр получившегося числа на 9 равен 7 (поэтому сумма цифр получившегося числа равна 9k + 7). Если зачеркнутая цифра равна x, то 9n + 3 - x = 9k + 7, отсюда x = 9(n - k) + 3 - 7, то есть зачеркнутая цифра равна 9 + 3 - 7 = 5.",5,msu,11 +"При каком наименьшем натуральном k выражение 2017 * 2018 * 2019 * 2020 + k является квадратом натурального числа? +","Докажем, что уже k = 1 подходит. Пусть n = 2018, тогда при k = 1 выражение из условия равняется (n − 1)n(n + 1)(n + 2) + 1 = (n − 1)(n + 2) * n(n + 1) + 1 = (n**2 + n − 2)(n**2 + n) + 1 = ((n**2 + n − 1) − 1)((n**2 + n − 1) + 1) + 1 = (n**2 + n − 1)**2",1,ommo,11 +"В школе имеется три кружка: по математике, по физике и по информатике. Директор как-то заметил, что среди участников кружка по математике ровно 1/6 часть ходит ещё и на кружок по физике, а 1/8 часть — на кружок по информатике; среди участников кружка по физике ровно 1/3 часть ходит ещё и на кружок по математике, а ровно 1/5 — на кружок по информатике; наконец, среди участников кружка по информатике ровно 1/7 часть ходит на кружок по математике. А какая часть участников кружка по информатике ходит на кружок по физике?","Пусть участников кружка по информатике x; тогда детей, которые ходят одновременно на кружок по математике и информатике x/7; тогда участников кружка по математике 8x/7, а детей, которые ходят одновременно на кружок по математике и по физике — 4x/21; тогда участников кружка по физике 4x/7, а детей, которые ходят одновременно на кружок по информатике и по физике — 4x/35",4/35,ommo,11 +Что больше: 1 или 21/64 + 51/154 + 71/214?,21/64 + 51/154 + 71/214 < 21/63 + 51/153 + 71/213 = 1/3 + 1/3 + 1/3 = 1.,1,ommo,11 +"В футбольном турнире играли семь команд: каждая команда по одному разу сыграла с каждой. В следующий круг отбираются команды, набравшие тринадцать и более очков. За победу даётся 3 очка, за ничью — 1 очко, за поражение — 0 очков. Какое наибольшее количество команд может выйти в следующий круг?","Всего командами сыграна 7 * 6/2 = 21 игра, в каждой из которых разыгрывалось 2 или 3 очка. Следовательно, максимальное количество очков, которое суммарно может быть у всех команд это 21 * 3 = 63. Значит, количество вышедших в следующий этап команд n удовлетворяет неравенству n * 13 <= 63, откуда n <= 4.",4,ommo,11 +Представьте в виде несократимой дроби 7 * 19/2015 * 6 * 19/2016 − 13 * 1996/2015 * 2 * 1997/2016 − 9 * 19/2015 .,"После замены смешанных дробей на обыкновенные, получаем, что исходное выражение равно 14124/2015 * 12115/2016 − 28191/2015 * 6029/2016 − 171/2015 = (171112260 − 169963539 − 344736)/(2015 * 2016) = 803985/(2015 * 2016) = (2015 * 21 * 19)/(2015 * 21 * 96) = 19/96 .",19/96,ommo,11 +"Федерация спортивной борьбы присвоила каждому участнику соревнования квалификационный номер. Известно, что во встречах борцов, квалификационные номера которых отличаются более, чем на 2 номера, всегда побеждает борец с меньшим номером. Турнир для 256 борцов проводится по олимпийской системе: в начале каждого дня бойцы разбиваются на пары, проигравший выбывает из соревнований (ничьих не бывает). Какой наибольший квалификационный номер может иметь победитель?","Заметим, что борец с номером k может проиграть только борцу с номером k+1 или k+2, поэтому после каждого тура наименьший номер не может увеличиться больше, чем на 2 номера. На турнире с 256 участниками 8 туров, следовательно, номер победителя турнира не превосходит 1 + 2 * 8 = 17. Предположим, что борец с номером 17 может победить. Тогда в первом туре должны выбыть борцы с номерами 1 и 2. Это возможно только если борец с номером 1 проиграл борцу с номером 3, а борец с номером 2 проиграл ��орцу с номером 4. Значит после первого тура борцы с номерами 3 и 4 останутся. Аналогично, после второго тура останутся борцы с номерами 5 и 6, после третьего — 7 и 8, . . . , после седьмого — 15 и 16. Значит в последнем, финальном, бою встретятся борцы с номерами 15 и 16. Противоречие с предположением, что борец с номером 17 может победить. Покажем, что борец с номером 16 может победить. Назовём борцов с номерами большими 16 слабыми. Пусть в туре с номером k <= 7 борец с номером 2k − 1 проиграет борцу с номером 2k + 1, борец с номером 2k проиграет борцу с номером 2k + 2, борцы с номерами 2k + 3, . . . , 16 победят каких-то слабых борцов, оставшиеся слабые борцы как-то сыграют между собой. Тогда после 7 туров останутся борцы с номерами 15 и 16, и в финальном бое борец с номером 16 победит",16,ommo,11 +Сумма первых тринадцати членов некоторой арифметической прогрессии составляет 50% от суммы последних тринадцати членов этой прогрессии. Сумма всех членов этой прогрессии без первых трёх относится к сумме всех членов без последних трёх как 4/3. Найти количество членов этой прогрессии.,"Обозначим через a первый член арифметической прогрессии, через d — ее разность, через n — количество членов. Тогда, сумма первых тринадцати ее членов будет равна 13 * (a + (a + 12d))/2 , последних тринадцати — (13 * (a + (n − 13) d + a + (n − 1)d))/2 , всех без первых трех — ((n − 3) * (a + 3d + a + (n − 1)d))/2 , всех без последних трех — ((n − 3) * (a + a + (n − 4)d))/2 . Из условия тогда имеем систему: 2 * 13(a + 6d) = 13(a + (n − 7)d), 3 * (n − 3) * (2a + (n + 2)d) = 4 * (n − 3) * (2a + (n − 4)d) , или, после преобразований, a = (n − 19)d, −2a = (n − 22)d Умножая первое равенство на 2 и прибавляя ко второму, получаем (3n − 60)d = 0. Поскольку d != 0 (так как иначе сумма всех членов без первых трех, равнялась бы сумме всех членов без последних трех), получаем n = 20.",20,ommo,11 +"Четырехзначное число X не кратно 10. Сумма числа X и числа, записанного теми же цифрами в обратном порядке, равна N. Оказалось, что число N делится на 100. Найдите N.","Пусть X = 1000a + 100b + 10c + d, Y = 1000d + 100c + 10b + a, при этом a, b, c, d — цифры и a != 0. По условию X + Y делится на 100, т.е. 1001(a + d) + 110(b + c) . . . 100. Имеем 1001(a + d) . . . 10, т.е. a + d . . . 10, откуда, поскольку a и d — цифры и a != 0, 1 <= a + d <= 18, значит a + d = 10. Далее, 1001 * 10 + 110(b + c) . . . 100, т.е. b + c + 1 . . . 10, откуда, поскольку b и c — цифры, 1 <= b + c + 1 <= 19, значит, b + c = 9. Таким образом, N = X + Y = 1001 * 10 + 110 * 9 = 11000.",11000,ommo,11 +В конус вписан цилиндр объема 9. Плоскость верхнего основания этого цилиндра отсекает от исходного конуса усеченный конус объемом 63. Найдите объем исходного конуса.,"Пусть высота и радиус исходного конуса равны H и R, а высота и радиус цилиндра равны h и r. Воспользуемся формулой для объема усеченного конуса: 1/3π(R**2 +Rr +r**2)h = 63. Также мы знаем, что πr**2h = 9. Поделив соответствующие части равенств получаем (R/r)**2 + (R/r) + 1 = 63 * 3/9 = 21. Решая квадратное уравнение получаем корни 4 и −5, геометрический смысл имеет только положительный. R/r = 4, (H−h)/H = 4, h/H = 3/4 , откуда получаем для исходного конуса: V = 1/3πR**2H = 1/3(πr**2h)(R/r)**2 * (H/h) = 1/3 * 9 * 4**2 * 4/3 = 64",64,ommo,11 +"Пройдя 2/5 длины узкого моста, пешеход заметил, что сзади к мосту приближается машина. Тогда он пош¨ел назад и встретился с машиной у начала моста. Если бы пешеход продолжал идти вперед, то машина догнала бы его у конца моста. Найти отношение скорости машины к скорости пешехода.","За время t, которое пешеход двигался навстречу машине до встречи у начала моста, он прош¨ел 2/5 длины моста. Следовательно, если бы пешеход продолжал идти впер¨ед, то за время t он прош¨ел бы ещ¨е 2/5 длины моста и ему осталось бы пройти 1/5 длины моста, а согласно условию, машина за время t подъехала бы к началу моста и до встречи с пешеходом ей осталось бы проехать мост целиком. Значит, отношение скорости машины к скорости пешехода равно 5.",5,msu,11 +Сколько решений имеет уравнение 1/(x − 1)**2 + 1/(x − 2)**2 = 2/x**2?,"Исходное уравнение при условиях x != 0, x != 1, x != 2 равносильно уравнению 6x**3 − 21x**2 + 24x − 8 = 0. Рассмотрим функцию f(x) = 6x**3 − 21x**2 + 24x − 8. Поскольку f′(x) = 18x**2 − 42x + 24, то x = 1 — точка максимума, а x = 4/3 — точка минимума. На области (−∞, 1) ∪ ( 4 3 ; +∞) функция f возрастает, на промежутке (1; 4/3) убывает. Так как f(0) = −8, f(1) = 1, f(4/3) = 8/9 , то уравнение f(x) = 0 имеет единственный корень, который лежит на промежутке (0; 1).",1,msu,11 +"В какую степень надо возвести корень x0 уравнения x**11 + x**7 + x**3 = 1, чтобы получить число x0 ** 4 + x0 ** 3 − 1?","Если x0 = 1, то и x0 ** 4 + x0 ** 3 − 1 = 1, следовательно, в этом случае степень может быть любой. Но число x0 = 1 не удовлетворяет уравнению x**11 + x**7 + x**3 = 1, поэтому x0 != 1. Поскольку 1 = x0 ** 11 + x0 ** 7 + x0 ** 3, получаем x0 ** 4 + x0 ** 3 − 1 = x0 ** 4 + x0 ** 3 − x0 ** 11 − x0 ** 7 − x0 ** 3 = x0 ** 4 * (1 − x0 ** 7 − x0 ** 3) = x0 ** 4 * (x0 ** 11 + x0 ** 7 + x0 ** 3 − x0 ** 7 − x0 ** 3) = x0 ** 15.",15,msu,11 +"Маша выписала на доске подряд все натуральные числа от 2 до 2015. Пришёл Ваня и заменил каждое из этих чисел суммой его цифр. Пришла Таня и сделала то же самое с получившимися числами. Так продолжалось до тех пор, пока на доске не осталось 2014 однозначных чисел (цифр). Какова сумма всех оставшихся чисел?","Число a и сумма цифр числа a при делении на 9 дают одинаковые остатки, поэтому в итоге на доске останется ряд чисел: 2, 3, 4, 5, 6, 7, 8, 9, 1, 2, …, 9, 1, 2, и так далее. Так как 2014 = 9 * 223 / 7, то в этом ряду 223 раза встретится последовательность от 1 до 9 и будет ещё 7 цифр. Значит, ряд заканчивается цифрой 8, и искомая сумма чисел равна (1 + 2 + ... + 9) * 224 - 1 - 9 = 45 * 224 - 10 = 10070.",10070,msu,11 +"Для охраны объекта в течение 5 суток заказчик договорился с охранниками о следующем: все они укажут отрезки времени своих предполагаемых дежурств с единственным условием, чтобы их объединение составляло заданные 5 суток, а он выберет из этих отрезков любой набор, удовлетворяющий тому же условию, и оплатит работу из расчета 500 руб. в час каждому дежурному. Какая наименьшая сумма денег, заранее подготовленная заказчиком, позволит ему наверняка расплатиться с охранниками?","1. Двойной оплаты времени охраны точно хватит, поскольку из любого покрытия отрезка конечной системой отрезков можно выбрать не более чем двукратное подпокрытие (если какая-то точка покрыта более чем двумя отрезками, то можно оставить только два из них — с самым левым концом и с самым правым, а остальные выбросить). 2. Менее чем двойной оплаты может не хватить при грамотных действиях охранников: они могут поставить свои дежурства так, чтобы ни одного из них выбросить было нельзя, а суммарная длина дежурств была сколь угодно близкой к удвоенному периоду наблюдения: 2 * 5 * 24 * 500 = 120 тыс. руб.",120000,msu,11 +"Прямоугольная таблица состоит из 5681 одинаковой клетки. Петя и Вася пронумеровали клетки натуральными числами 1, 2, …, 5681 подряд. Петя нумеровал клетки по строкам слева направо (сначала первую строку, затем вторую и т. д.), а Вася – по столбцам сверху вниз (сначала первый столбец, затем второй и т. д.). Оказалось, что ровно в 5 клетках их номера совпали. Чему равна сумма числа строк и числа столбцов в этой таблице?","Пусть в таблице m строк и n столбцов, а клетка, получившая одинаковые номера, расположена в строке с номером i и в столбце с номером j. Тогда, если считать по строкам, в этой клетке стоит число (i - 1) * n + j, а если считать по столбцам, то это число равно (j - 1) * m + i. Следовательно, (i - 1) * n + j = (j - 1) * m + i и (i - 1) * (n - 1) = (j - 1) * (m - 1). Если m = 1 или n = 1 , то номера Пети и Васи совпадут во всех клетках. Значит, m > 1 и n > 1 . Пусть d = НОД(n - 1, m - 1), тогда n - 1 = pd , m - 1 = pd , где НОД(p, q) = 1. Получаем (i - 1) * p = (j - 1) * q. Поэтому i - 1 = qk, j - 1 = pk, k = 0, 1, ..., d. Следовательно, количество клеток, получивших одинаковые номера, равно d + 1 = НОД(n - 1, m - 1) + 1 . Так как 5681 = 13 * 19 * 23, то n =13, m = 19 * 23 = 437 или, наоборот, n = 437 , m =13 . В любом случае m + n = 450.",450,msu,11 +Решите уравнение 1 – (2 – (3 – (...2010 – (2011 – (2012 – x))...))) = 1006.,"Открыв скобки, получим 1 – 2 + 3 – 4 + … +2011 – 2012 + x = 1006; -1006 + x = 1006; x=2012.",2012,school,10 +Дорогу длиной 28 километров разделили на три неравные части. Расстояние между серединами крайних частей равно 16 км. Найдите длину средней части.,"Расстояние между серединами крайних частей складывается из половин крайних участков и целого среднего участка, т.е. удвоенное это число равно длине дороги плюс длина среднего участка. Т.о. длина среднего участка = 16 * 2 - 28 = 4.",4,school,10 +Решите числовой ребус: ТЭТА + БЭТА = ГАММА. (Разные буквы – разные цифры.),"Так как A + A заканчивается на А, то А = 0. Т.к. Г – результат переноса в следующий разряд, то Г = 1. Так как A + A заканчивается на А, то А = 0. Значит переноса в разряд десятков нет, т.е. Т + Т заканчивается на М, и значит М – четно. Переноса в разряд сотен тоже нет, т.к. иначе нечетное число Э + Э + 1 заканчивалось бы на четное М. Т.к. переноса нет, то 2ТБ < 10. Возможные варианты 2, 3, 4. Если Т = 2, то Э = 7, откуда Б = 7 – но 7 уже занята. Если Т = 3, то М = 6, Э = 8, откуда Б = 6, но 6 = М. И последний вариант Т = 4. Тогда М = 8, Э = 9. Откуда Б = 5 – противоречия нет. Таким образом, возможен только один вариант: 4940 + 5940 = 10880",10880,school,10 +"Саша, Лё��а и Коля одновременно стартуют в забеге на 100 м. Когда Саша финишировал, Лёша находился в десяти метрах позади него, а когда финишировал Лёша — Коля находился позади него в десяти метрах. На каком расстоянии друг от друга находились Саша и Коля, когда Саша финишировал? (Предполагается, что все мальчики бегут с постоянными, но, конечно, не равными скоростями.)","Скорость Коли составляет 0,9 от скорости Лёши. В момент, когда Саша финишировал, Лёша пробежал 90 м, а Коля 0,9 * 90 = 81 м. Следовательно, расстояние между Сашей и Колей было 19 м",19,school,8 +"Каждый из 10 гномов либо всегда говорит правду, либо всегда лжет. Известно, что каждый из них любит ровно один сорт мороженого: сливочное, шоколадное или фруктовое. Сначала Белоснежка попросила поднять руки тех, кто любит сливочное мороженое, и все подняли руки, потом тех, кто любит шоколадное мороженое – и половина гномов подняли руки, потом тех, кто любит фруктовое мороженое – и руку поднял только один гном. Сколько среди гномов правдивых?","Гномы, которые всегда говорят правду, подняли руку один раз, а гномы, которые всегда лгут, – два раза. Всего было поднято 16 рук (10 + 5 + 1). Если бы все гномы сказали правду, то было бы поднято 10 рук. Если одного правдивого гнома заменить на одного лгуна, то число поднятых рук увеличится на 1. Так как было поднято 6 «лишних» рук, то 6 гномов солгали, а 4 сказали правду.",4,school,8 +"Назовем число зеркальным, если слева направо оно «читается» так же, как справа налево. Например, число 12321 – зеркальное. Сколько существует пятизначных зеркальных чисел, которые делятся на 5?","Число, которое делится на 5, должно оканчиваться на 5 или на 0. Зеркальное число оканчиваться на 0 не может, так как тогда оно должно на 0 начинаться. Итак, первая и последняя цифры - это 5. Вторая и третья цифра могут быть любыми – от сочетания 00 до сочетания 99 – всего 100 вариантов. Так как четвертая цифра повторяет вторую, всего различных чисел будет 100.",100,school,8 +"Фирма изготавливает лимонный напиток, разбавляя лимонный сок водой. Сначала фирма производила напиток, содержащий 15% лимонного сока. Через некоторое время генеральный директор отдал указание снизить содержание лимонного сока до 10%. На сколько процентов увеличится количество производимого лимонного напитка при тех же объёмах поставок лимонов? ","Содержание лимонного сока в напитке после указания генерального директора снизилось в полтора раза. Значит, из тех же лимонов можно приготовить в полтора раза больше лимонного напитка. Иными словами, количество производимого лимонного напитка увеличится в полтора раза или на 50%.",50,school,8 +"Все натуральные числа, сумма цифр в записи которых делится на 5, выписывают в порядке возрастания: 5, 14, 19, 23, 28, 32, … Чему равна самая маленькая положительная разность между соседними числами в этом ряду? Приведите пример и объясните, почему меньше быть не может.","Раз��ость меньше 1 быть не может, так речь идет про разность различных натуральных чисел, например, между числами 49999 и 50000. Понятно, что если два соседних числа отличаются только в разряде единиц, то разность между ними равна 5 (например, 523 и 528). Значит, нужно, чтобы числа отличались и в других разрядах. +Можно попробовать взять большее число круглым, тогда числа будут отличаться минимум в двух +разрядах. Возьмем, например, 50, предыдущее число 46, а разность равна 4. Если взять 500, то +предыдущее число 497 и разность равна 3. Осталось подобрать такое число нулей, чтобы разность +была равна 1. +",1,school,8 +"В тот день, когда Диму поздравляли с днём рождения его брат и сестра, Дима сказал: «Смотрите, как интересно, я теперь вдвое старше брата и втрое старше сестры!» – «А ваш средний возраст 11 лет», – подхватил папа. Сколько лет исполнилось Диме?","Если возрасты Димы, его брата и сестры изобразить отрезками, то «Димин отрезок» состоит из двух «отрезков брата» или трех «отрезков сестры». Тогда, если возраст Димы поделить на 6 частей, то возраст сестры – две такие части, а возраст брата – три такие части. Тогда сумма их возрастов – 11 таких частей. С другой стороны, если средний возраст равен 11 лет, то сумма возрастов – 33 года. Откуда следует, что в одной части – три года. Значит, Диме исполнилось 18 лет.",18,school,9 +"Сколько существует трёхзначных чисел, которые в 5 раз больше произведения своих цифр?","В составе цифр, которыми записывается число, нет цифры 0, иначе не может быть выполнено условие задачи. Данное трехзначное число получено умножением на 5 произведения своих цифр, следовательно, оно делится на 5. Значит, его запись оканчивается цифрой 5. Получаем, что произведение цифр, умноженное на 5, должно делиться на 25. Заметим, что четных цифр в записи числа быть не может, иначе произведение цифр было бы равно нулю. Таким образом, трехзначное число должно делиться на 25 и не содержать четных цифр. Таких чисел только пять: 175, 375, 575, 775 и 975. Произведение цифр искомого числа должно быть меньше 200, иначе, умноженное на 5, даст четырехзначное число. Поэтому числа 775 и 975 заведомо не подходят. Среди оставшихся трех чисел только 175 удовлетворяет условию задачи.",1,school,9 +"За круглым столом сидят 30 человек — рыцари и лжецы (рыцари всегда говорят правду, а лжецы всегда лгут). Известно, что у каждого из них ровно один друг, причем у рыцаря этот друг — лжец, а у лжеца этот друг — рыцарь (дружба всегда взаимна). На вопрос «Сидит ли рядом с вами ваш друг?» сидевшие через одного ответили «да». Сколько из остальных могли также ответить «да»? (Перечислите все варианты и докажите, что других нет.)","Из условия следует, что все сидящие за столом разбиваются на пары друзей; значит, рыцарей и лжецов поровну. Рассмотрим любую пару друзей. Если они сидят рядом, то рыцарь на заданный вопрос ответит «да», а лжец — «нет». Если же они не сидят ряд��м, то их ответы будут противоположными. В любом случае ровно один из пары друзей даст ответ «да». Значит, при любой рассадке все остальные 15 ответов бу- дут «нет».",0,region,9 +"На окружности отмечено 2N точек ( N — натуральное число) Известно, что через любую точку внутри окружности проходит не более двух хорд с концами в отмеченных точках. Назовем паросочетанием такой набор из N хорд с концами в отмеченных точках, что каждая отмеченная точка является концом ровно одной из этих хорд. Назовём паросочетание чётным, если количество точек, в которых пересекаются его хорды, чётно, и нечётным иначе. Найдите разность между количеством чётных и нечётных паросочетаний.","Приведём другое доказательство шага индукции. Пусть отмеченные точки — A1,... ,A2N . Рассмотрим все паросочетания, в которых A2N − 1 и A2N соединены хордой. Эта хорда не пересекается ни с одной другой. Значит, выбросив её из каждого из рассматриваемых паросочетаний, мы получим все паросочетания на точках A1,... ,A2N − 2, причём чётность каждого из них сохранится. По предположению индукции, среди на- ших паросочетаний чётных на одно больше, чем нечётных. Для завершения доказательства достаточно показать, что среди всех остальных паросочетаний поровну чётных и нечётных. Рассмотрим любое из них; пусть в нём есть хорды A2N−1Ai и A2N Ak. Теперь «поменяем местами» точки A2N − 1 и A2N , то есть заменим наши хорды на A2N Ai и A2N − 1Ak. При этом, если исходная хорда пересекалась с какой-то из остальных, то и новая хорда будет с ней пересекаться. С другой стороны, если хорды A2N − 1Ai и A2N Ak не пересекались, то новые хорды будут пересекаться, и наоборот. Итак, каждому оставшемуся чётному паросочетанию мы сопоставили нечётное, и наоборот; при этом разным паросочетаниям, очевидно, соответствуют разные. Значит, оставшихся чётных и нечётных паросочетаний поровну, что и требовалось доказать.",1,region,10 +"Петя выбрал натуральное число a > 1 и выписал на доску пятнадцать чисел 1 + a, 1 + a**2 , 1 + a**3 , . . . , 1 + a**15. Затем он стёр несколько чисел так, что любые два оставшихся числа взаимно просты. Какое наибольшее количество чисел могло остаться н а доске?","Покажем сначала, что искомых чисел не может быть более четырех. Заметим, что если k — нечётное, то число 1 + a**(nk) = 1**k + (a**n)**k делится на 1 + a**n. Далее, каждое из чисел 1, 2, . . . , 15 имеет один из видов k, 2k, 4k, 8 k, где k нечётно. Таким образом, каждое из чисел 1 + a, 1 + a 2 , 1 + a 3 , . . . , 1 + a15 делится либо на 1 + a, либо на 1 + a**2, либо на 1 + a**4, либо на 1 + a**8. Поэтому, если мы возьмем хотя бы пять чисел, то среди них найдутся два, делящиеся на одно и то же число, большее 1; значит, они не будут взаимно просты. Итак, оставшихся чисел не более четырех. Осталось показать, что четыре числа могли остаться. Действительно, если a = 2, то можно оставить числа 1 + 2 = 3, 1 + 2**2 = 5, 1 + 2**4 = 17 и 1 + 2**8 = 257. Все они попарно взаимно просты.",4,region,10 +"2011 складов соединены дорогами так, что от любого с��лада можно проехать к любому другому, возможно, проехав по нескольким дорогам. На складах находится по x1, . . . , x2011 кг цемента соответственно. За один рейс можно провезти с произвольного склада на другой склад по соединяющей их дороге произвольное количество цемента. В итоге на складах по плану должно оказаться по y1, . . . , y2011 кг цемента соответственно, причём x1 + x2 + . . . + x2011 = y1 + y2 + . . . + y2011. За какое минимальное количество рейсов можно выполнить план при любых значениях чисел xi и yi и любой схеме дорог?","Покажем вначале, что за 2009 рейсов план выполнить удастся не всегда. Пусть (при произвольной схеме дорог) изначально весь цемент расположен на одном складе S, а распределить его нужно по всем складам поровну. Тогда на каждый склад, кроме S, нужно в каком-нибудь рейсе цемент завезти; ясно, что такие 2010 рейсов различны, поэтому всего рейсов должно быть не меньше 2010. Нам осталось показать, что за 2010 рейсов план всегда удастся выполнить. Мы докажем индукцией по n, что при n складах всегда удастся обойтись n − 1 рейсом. База при n = 1 очевидна. Пусть n > 1. Так как с любого склада можно добраться до любого другого, то существует маршрут, проходящий по всем складам (может быть, неоднократно). Рассмотрим любой такой маршрут и склад A, который впервые появился на этом маршруте позже всего. Тогда, если удалить склад A и все дороги, ведущие из него, то по-прежнему от любого склада до любого другого можно добраться (по предыдущим дорогам маршрута). Можно считать, что A — склад с номером n. Если yn <= xn, то вывезем из A на любой соединённый с ним склад xn − yn кг цемента, а после этого забудем про него и про все дороги, из него ведущие. По предположению индукции, для оставшихся складов можно выполнить план за (n − 1) − 1 рейс. В итоге через (n − 2) + 1 рейс получится требуемое распределение цемента. Если же yn > xn, то мы уже доказали, что из распределения, когда на i-м складе находится yi кг, можно получить распределение, когда на i-м складе находится xi кг, за n − 1 рейс. Проведя теперь все эти перевозки в обратном порядке (и обратном направлении), мы осуществим требуемый план.",2010,region,11 +"Квадратный трехчлен f(x) таков, что многочлен (f(x))**5 − f(x) имеет ровно три вещественных корня. Найдите ординату вершины графика этого трехчлена.","Так как g(x) = (f(x))**5− f(x) = f(x) * (f(x) − 1) * (f(x) + 1) * ((f(x))**2 + 1), то корнями нашего многочлена являются корни трехчленов f(x), f(x) − 1 и f(x) + 1 (поскольку многочлен (f(x))**2 + 1 всюду положителен). Ясно, что любое число может быть корнем только одного из них. Пусть y0 — искомая ордината вершины. Предположим, что y0 != 0. Будем считать, что старший коэффициент в f(x) положителен (иначе заменим f(x) на − f(x), при этом y0 заменится на − y0). Предположим, что y0 > 0; тогда f(x) > 0 и f(x) + 1 > 0 при всех x, значит, корни многочлена g(x) являются корнями f(x) − 1, а их не больше двух. Если же y0 < 0, то трехчлены f(x) и f(x) − 1 имеют по два корня, значит, g(x) имеет хотя бы 4 ко��ня. Оба случая невозможны; значит, y0 = 0.",0,region,11 \ No newline at end of file